Menu Expand
USMLE Step 1 Secrets E-Book

USMLE Step 1 Secrets E-Book

Thomas A. Brown | Sonali J Bracken

(2012)

Additional Information

Book Details

Abstract

Use the proven format of the best-selling Secrets SeriesTM to prepare for the USMLE Step 1. Presented as questions and answers based on clinical vignettes, USMLE Step 1 Secrets will aid you in not only retaining but understanding key information for the boards. Co-authored by Dr.Tom Brown, author of Rapid Review in Physiology, and Dr.Dave Brown, the second edition has been updated to be even more concise and ultra high-yield, while still offering a complete overview of the key topics and the top “secrets” and personal insights from the authors.

  • Utilizes a case-based approach to prepare you for the Step 1 exam.
  • Implements systems-based organization for an integrated approach.
  • Uses the Secrets Q&A format to make learning interesting and effective.
  • Uses a new Top 100 Secrets chapter as a recap of the most important information in the book to help you review quickly.
  • Incorporates 7 new chapters, including Clinical Anatomy and Pharmacology, and Toxicology to expand coverage of high-yield areas and keep you current.
  • Provides maximum information in minimal time to help fit a concise review into your busy schedule.
  • Includes a color pathology and dermatology section to enhance the clarity and understanding of key images.

Table of Contents

Section Title Page Action Price
Front Cover Cover
Usmle Step 1 Secrets iii
Copyright iv
Contributors v
Preface vii
More Secrets for Success on the Usmle Step 1 xi
Contents xv
Chapter 1: Cardiology 1
Basic concepts-hemodynamics 1
1. What are the mathematical determinants of the arterial blood pressure? 1
2. What are the primary determinants of cardiac output? 1
3. What are the three main factors that affect stroke volume? 2
4. What is preload, and how does it affect stroke volume? 2
5. What is contractility and how does it affect stroke volume? 2
6. What is afterload and how does it affect stroke volume? 3
7. What are the primary determinants of peripheral resistance? 3
8. What is the mechanism by which the sympathetic nervous system responds to a reduction in blood pressure? 3
9. How do the α1-receptor antagonists work? 4
10. How do the α1-receptor antagonists cause orthostatic hypotension? 4
11. What hemodynamic changes occur during exercise? 4
12. Which clinical scenarios would shift the CO and venous return curves to the points labeled 1 to 4 on Figure1-3? 4
Basic concepts-excitation-contraction coupling 5
1. What is the source of cytosolic calcium during ventricular systole? 5
2. What is the function of calcium in cardiac muscle contraction? 6
3. What is the mechanism by which β-adrenergic stimulation increases cardiac contractility? 6
4. What is the contribution of the sympathetic nervous system to ventricular relaxation? 6
Basic concepts-arrhythmias 6
1. What is the relationship between the various phases of the ventricular myocyte action potential and the different ion flux 6
2. What is responsible for the drifting of the resting membrane potential in nodal cells? 7
3. Through what mechanism does sympathetic stimulation increase heart rate? 7
4. What are the classes of antiarrhythmics and how do their mechanisms of action and potential side effects vary? 8
Case 1-1 9
1. What are the types of hypertension and which does this patient most likely have? 9
2. How is hypertension defined and what are the potential complications? 9
3. What are four classes of drugs that could be useful in treating this man's hypertension? 10
4. What is the mechanism by which metoprolol will reduce blood pressure in this man? 10
5. What pharmacologic property makes certain beta blockers more ``cardioselective´´ than others? 10
6. Caution should be used in prescribing beta blockers for patients with which comorbid conditions and why? 10
7. What might be occurring in this patient and how would you recommend he discontinue his medication? 11
Case 1-2 12
1. How would the fact that this patient also has diabetes influence treatment for his hypertension? Consider drug choice and 12
2. What are the mechanisms by which ACE inhibitors lower blood pressure? 12
3. Why might the use of beta blockers to control this man's hypertension not be an ideal choice 13
4. What effect might starting a thiazide diuretic have on glycemic control in this patient? 13
Case 1-2 continued: 13
5. What is the mechanism by which ACE inhibitors, such as lisinopril, can cause a cough? 13
6. Cover the columns on the right side of Table1-3, and for each antihypertensive drug in the left column, try to name the 13
Case1-3 15
1. What is the likely diagnosis? 15
2. Does this patient have stable or unstable angina? How are they different, and which is more serious? 16
3. How can angina occur in the absence of coronary atherosclerosis? 16
4. What are the principal physiologic determinants of myocardial oxygen supply? 16
5. What are the principal physiologic determinants of myocardial oxygen demand? 17
6. Which factors contribute to a myocardial oxygen demand that exceeds supply in this man? 17
7. Why is nitroglycerin effective in eliminating anginal pain? 17
8. What are the mechanisms by which beta blockers decrease myocardial oxygen demand and, therefore, the symptoms of angina? 17
Case 1-3 continued: 18
9. How could a similar evaluation be made in a patient in whom an exercise stress is contraindicated (e.g., orthopedic condit 18
Case1-4 18
1. What disorder is on top of the differential diagnosis list at this point and why? 18
2. What are some common risk factors for myocardial infarction and cardiovascular disease in general? 18
3. What kind of myocardial infarction is this patient experiencing? 19
4. In addition to analysis of the electrocardiogram, what serum tests could be ordered to confirm 19
5. What other treatment option is available besides angioplasty to restore coronary blood flow? 19
6. What is the physiologic rationale for giving beta blockers to patients who have had heart attacks? 20
7. How might a myocardial infarction result in the following short- and long-term abnormalities? 20
8. What is sudden cardiac death, and why are patients who have had myocardial infarctions predisposed to it? 21
Case1-5 22
1. What disorder is at the top of your differential diagnosis list? 22
Case 1-5 continued: 22
2. In pathophysiologic terms, what is heart failure? 22
3. What are the differences between systolic and diastolic heart failure? Which does this woman most likely have? 22
4. What is the etiology of heart failure? 23
5. The body's response to heart failure is initially helpful but becomes maladaptive with time. 23
6. Given the pathophysiologic adaptations in heart failure just noted, why might beta blockers be beneficial in heart failure? 24
7. How is digitalis, which is used in heart failure, believed to increase cardiac contractility? 24
8. In Table1-4, cover the right-hand column and attempt to give the mechanism of action for each of the listed drugs used. . . 25
Other Related Questions 25
9. What is ``high-output´´ heart failure? 25
Case1-6 26
1. What is the diagnosis? 26
2. What likely predisposed this patient to developing aortic stenosis? 26
3. What causes heart murmurs and why does this patient have one? 26
4. What compensatory left ventricular changes occur as a result of aortic stenosis? 27
5. Why does this patient have a weak, delayed carotid upstroke with a narrowed pulse pressure on physical 27
6. As a result of aortic stenosis, do ventricular myocytes spend more time in isotonic or isometric (isovolumic) contraction? 28
7. Explain the cause of this patient's presenting complaints. What is most likely causing his episodes of syncope? His chest... 28
8. What are some causes of increased myocardial oxygen demand in aortic stenosis? 28
9. What are some causes of decreased myocardial oxygen supply in aortic stenosis? 28
10. Why is atrial fibrillation a particularly dangerous complication in aortic stenosis (aside from the risk of embolic strok)? 28
11. Describe the murmur of aortic stenosis 29
Case1-7 30
1. This patient's constellation of symptoms is consistent with what syndrome, and what is its significance? 30
2. Part of this woman's presentation is dyslipidemia, a disorder of lipoprotein metabolism. 30
3. What are the functions of the various forms of lipoproteins? How are they removed from the circulation? 31
Case 1-7 continued:: 31
4. What is the mechanism of action of simvastatin (and other statins)? 31
5. In addition to liver tests, which other enzymes might be monitored when using a statin? 31
6. What are the cause and clinical significance of this woman's hypertriglyceridemia? 31
7. What enzymatic mechanism clears triglycerides from the circulation? 32
8. What is the clinical significance of elevations in LDL levels? 32
9. What is the clinical significance of this patient's low HDL-C? 32
10. What is the mechanism of action of the lipid-lowering fibrates/fibric acid derivatives (e.g., gemfibrozil, fenofibrate)? 32
11. How does niacin influence the lipid profile, and what is the unique pharmacologic feature of niacin 32
12. How do the bile-sequestering resins (e.g., cholestyramine, colestipol) lower LDL cholesterol levels? 33
Other Related Question 33
13. What is the cause of the disorder of lipid metabolism known as abetalipoproteinemia? 33
Chapter 2: Pulmonology 34
Basic concepts-mechanics of breathing 34
1. What are the driving forces for the following: 34
2. What are the forces of resistance for the following: 34
3. What does pulmonary compliance measure? 36
4. With respect to the compliance curve of the lungs, how might breathing at an elevated functional residual capacity in chro 36
5. What does pulmonary elastance measure? How is pulmonary elastance altered in restrictive and obstructive lung diseases and 36
6. How does surfactant affect alveolar surface tension? 37
7. Why are smaller alveoli more prone to collapse, and how is this relevant to neonatal respiratory distress syndrome? 37
8. What is the minute ventilation? 37
9. What is ``dead space´´? What is the difference between anatomic dead space and physiologic dead space? 38
10. Why is the alveolar minute ventilation a much better representation of functional ventilation? 38
Basic concepts-ventilation-perfusion matching 38
11. What does the ventilation/perfusion ratio measure, and what is its approximate value? What is an ``ideal´´ value for 38
12. What conditions cause an increase in the / ratio? 39
13. What causes a decrease in the ventilation/perfusion ratio? 39
14. What effect does chronic obstructive pulmonary disease usually have on the ventilation-perfusion ratio? 39
15. What is the difference between an anatomic shunt and a physiologic shunt? 39
Basic concepts-gas exchange 39
16. What influences the diffusion of gases from the alveoli into the pulmonary capillaries, and vice versa? 39
17. What is the alveolar-arterial oxygen gradient and what is the clinical significance of its magnitude? 40
18. How is the alveolar-arterial oxygen gradient calculated? 40
19. What are the primary determinants of respiratory drive? 40
20. How does increasing or decreasing the arterial Pco2 affect pH? 41
21. What is respiratory acidosis? Respiratory alkalosis? 41
22. What are the forced expiratory volume and the FEV1/FVC ratio? 41
23. What is the principal difference between ``restrictive´´ and ``obstructive´´ lung disease with respect to 41
Case2-1 42
1. What disorder do you suspect? 42
Case2-1 continued: 42
2. What is the diagnosis? How is this condition classified? 43
3. What is the pathophysiologic process causing symptoms in this woman? 43
4. About 5% of asthmatics are sensitive to aspirin, and some may even develop fatal bronchospasm from ingesting aspirin. W... 43
5. What is exercise-induced asthma? 43
6. What is the explanation for the decreased FEV1/FVC ratio in this woman? 43
7. Why does methacholine cause her FEV1/FVC ratio to decrease? 43
8. What is the residual lung volume and how is it affected in an asthma attack? 44
9. Why is wheezing generally heard most during expiration in asthma? 44
10. With respect to the pathophysiology of asthma, what are the two mechanistic targets of pharmacologic intervention? 44
11. Quick review: Cover the two columns on the right side of Table2-2 and identify the class of drug and mechanism of 44
12. Why are albuterol and salmeterol preferable to isoproterenol in treating asthma? 44
13. This patient was put on a combination inhaler that contained a β2-agonist and a corticosteroid. A month or so later she n 45
14. Why are cromolyn sodium and nedocromil useful in the prevention but not treatment of asthmatic attacks? 45
15. What histologic changes would be expected in the bronchial smooth muscle and mucosa if a biopsy were performed in this pa 45
16. What is theophylline and what are some of the drawbacks of its use? 45
Case2-2 46
1. What are some potential causes of this man's symptoms? 46
Case2-2 continued: 46
2. What is the most likely diagnosis at this time? 46
3. What is the pathophysiology of this disorder and why doesn't albuterol help? 46
4. How is hypersensitivity pneumonitis treated? 46
Case2-3 47
1. What is the suspected diagnosis? 47
2. What other tests can be done to more confidently establish this diagnosis? 47
3. What is causing the pulmonary infiltrate? 47
Case2-4 47
1. What is the most likely diagnosis in this patient? 48
2. How would your differential diagnosis change if the patient presented with a 2-week history 48
3. What are some common causes of pneumonia in different age groups? 48
Case2-5 48
1. What is the differential diagnosis? 48
Case2-5 continued: 48
2. What is the diagnosis? 49
3. What is the pathophysiologic explanation for the decreased FEV1/FVC ratio in this woman? 49
4. What was the most likely cause of emphysema in this woman? 49
5. What histopathologic changes would a biopsy of this woman's lung likely reveal? 49
6. How do centriacinar and panacinar emphysema differ in terms of morphology and etiology? 50
7. Why may administration of large volumes of concentrated O2 to this woman cause hypoventilation? 50
8. How does emphysema cause the right ventricular hypertrophy seen on this patient's electrocardiogram? 50
Related Questions on Chronic Obstructive Pulmonary Disease 50
9. Why is an individual with α1-antitrypsin deficiency at increased risk for developing emphysema? 50
10. Why do patients with emphysema breathe through pursed lips? 50
11. What pharmacologic agents are used to treat chronic obstructive pulmonary disease? 51
Case2-6 52
1. What is the likely diagnosis? 52
2. Why may cigarette smoking predispose to chronic lung infections? 52
Case2-6 continued: 52
3. What is the pathogenesis of chronic bronchitis? 52
4. What is the basis of the reduced FEV1/FVC ratio in chronic bronchitis? 52
5. What is an acute exacerbation of chronic obstructive pulmonary disease? What classes of medications should be avoided or p 52
6. What acid-base abnormality is commonly found in chronic obstructive pulmonary disease patients and what is its origin? 53
7. How might mechanical ventilation lead to respiratory alkalosis? 53
Case2-7 54
1. What is the likely diagnosis? 54
Case2-7 continued: 54
2. What is the diagnosis? 54
3. What are asbestos, asbestosis, and the pneumoconioses? 54
4. To which malignancies does asbestosis predispose? 55
5. What are some other common causes of pulmonary fibrosis? 55
6. What is the pathophysiologic explanation for the reduction in forced vital capacity in this man? 56
7. What lung biopsy finding may be present in this patient that is unique to asbestosis? 56
Case2-8 56
1. What is the likely cause of this woman's dyspnea? 56
2. What is the difference between the Pao2 and the arterial oxygen content? 56
Case2-8 continued: 57
3. What does arterial oxygen saturation measure? 57
4. Why doesn't this woman's anemia make her cyanotic? 57
5. How is the anemia contributing to this woman's dyspnea? 57
Related Questions 58
6. What are typical values for arterial and venous Po2 and what do they represent with respect to the hemoglobin dissociation 58
7. Would exposure to carbon monoxide be expected to affect the Pao2? Explain. 58
8. Differentiate between external and internal respiration. Which is affected by anemia? 59
Case2-9 60
1. What is the anticipated diagnosis? 60
2. What are the etiology and pathogenesis of this disease? 60
3. How is this condition managed? 60
4. What is the diffusion equation and which parameter is influenced most by the high concentration of inspired oxygen? 60
5. Both congestive heart failure and acute respiratory distress syndrome can cause significant pulmonary edema. How does the 60
6. Why is it necessary to be particularly cautious in giving fluids to this patient? 60
Related Question 61
7. Using the diffusion equation, explain why each of the conditions in Table2-6 is associated with hypoxemia: 61
Case2-10 61
1. What type of lung cancer does this woman likely have? 61
2. If a patient with small cell carcinoma develops bilateral ptosis (droopy eyelids) as well as neuromuscular weakness, what 62
3. If a lung tumor is growing at the apex of the lung and compressing the cervical sympathetic chain on that side, what manif 62
4. Although there are several different histologic types of lung cancer, what are the two principal classifications and why d 63
5. Why are radon levels routinely measured before homes are purchased? 63
6. How can squamous cell carcinoma cause hypercalcemia without any bony metastases? 63
Differential Diagnosis 63
7. If a patient with small cell carcinoma has hypertension, hypernatremia, hypokalemia, abdominal striae, and a ``buffalo hu 63
Case2-11 63
1. What must be included in the differential diagnosis? 63
Case2-11 continued: 64
2. Now what is the most likely diagnosis? 64
3. Why was the lymph node biopsy performed? 64
4. How can sarcoidosis cause cor pulmonale (right ventricular failure)? 64
5. How is pulmonary hypertension defined? 64
6. What is the difference between primary and secondary pulmonary hypertension? 64
7. How can sarcoidosis cause a restrictive cardiomyopathy? 64
Case2-12 65
1. What is the diagnosis? 65
2. Why should the chest tube be inserted immediately superior to the lower rib in the intercostal space in which it is insert 65
3. What is the pressure inside the pleural cavity (intrapleural space) normally? 65
4. How does hypoxia-induced vasoconstriction help compensate, to some extent, for the respiratory dysfunction caused by pneum 65
5. Is this patient more likely to be experiencing respiratory acidosis or respiratory alkalosis? Explain. 66
6. What effect will a pneumothorax have on the serum ionized calcium level? 66
Chapter 3: Nephrology 67
Clinical renal disease 67
Case3-1 67
1. What is the probable cause of this patient's problems? 67
2. What are the three etiologic classifications of acute renal failure? 68
3. How is glomerular filtration rate (GFR) calculated? 68
4. Why is the absence of hematuria on urinalysis important in establishing the diagnosis? 68
5. Why was it important to ask this patient about tricyclic antidepressants, antipsychotics, antihistamines, and sympathomime 69
Case3-2 69
1. What is the probable diagnosis? 69
2. What is the pathophysiology of acute tubular necrosis? 69
3. What is the cause of the decreased glomerular filtration rate and oliguria seen in acute tubular necrosis? 70
4. What are the three phases of acute tubular necrosis? 70
5. Other than the presence of ``muddy brown´´ casts, how can prerenal azotemia due to ischemia be differentiated from i 70
Related Question 71
6. What is rhabdomyolysis and how can it cause acute tubular necrosis? 71
Case3-3 71
1. What is the most likely cause for her acute renal failure? 72
2. What are the three major types of NSAID-induced renal toxicity? 72
Case3-4 73
1. What is most likely causing her blood urea nitrogen and creatinine elevation? 73
2. What is the value of the normal fasting glucose and hemoglobin A1c levels in the differential diagnosis? 73
3. What is the difficulty in establishing that this patient's renal failure was definitely due to hypertension 74
4. How does this woman's renal failure explain her hypocalcemia? 74
5. Would parathyroid hormone levels be increased or decreased in this patient? 74
6. How does parathyroid hormone normally act to regulate serum Ca2+? How are parathyroid hormone levels normally regulated? 74
7. What are the potential pathologic manifestations of the hyperparathyroidism that develops in renal failure? 75
8. Why is this patient also predisposed to osteomalacia? 76
9. How could this woman's renal failure explain the following findings? 76
10. Is this woman suffering from azotemia or uremia? 76
11. If she were uremic, and a friction rub was detected on physical examination, what might you suspect? 76
Case3-5 77
1. What is the diagnosis? 77
2. What is the primary complication of this disease? 77
3. Why must urinary tract infections be treated aggressively in patients with this disease? 77
4. If this patient suddenly develops a severe headache, what vascular abnormality must be suspected? 77
5. How does autosomal dominant polycystic kidney disease differ from autosomal recessive polycystic kidney 78
6. What is tuberous sclerosis and how can it be differentiated from ADPKD? 78
7. What is von Hippel-Lindau syndrome and how can it be differentiated from ADPKD? 79
8. What is medullary cystic disease and how can it be differentiated from ADPKD? 79
9. Quick review: Cover the far right column in Table3-1 and give the characteristic features of each of the cystic... 79
10. Why is medullary sponge kidney not included in Table 3-1? 79
11. What is the most common cause of renal cysts? 79
Case3-6 81
1. What is the likely diagnosis in this child and why? 81
2. What is the likely cause of nephrotic syndrome in this patient? 81
3. Assuming minimal change disease as the underlying pathologic condition, what would you expect gross histologic examinatio... 81
4. Why might this boy be susceptible to infections while suffering from this illness? 82
5. How should this boy be managed? Should a renal biopsy be performed? 82
6. What are other major causes of nephrotic syndrome? 82
Some Differential Diagnosis Concepts 86
7. What condition might you suspect in a 6-year-old girl who presents with abdominal pain, joint pain, hematuria (or melena) 86
Case3-7 86
1. What is the likely diagnosis? 87
2. What segment of the nephron is involved in acute nephritis? How does this compare with the nephrotic syndrome? 87
3. What was the likely cause of this man's sore throat and what is its relationship to the renal dysfunction? 87
4. If the patient had recently suffered from cellulitis rather than pharyngitis, would it alter the diagnosis of his renal di 88
5. What is the treatment for poststreptococcal glomerulonephritis? 88
6. How would the diagnosis change if a renal biopsy revealed ``glomerular crescents´´? 88
7. What is the most common cause of nephritis worldwide? 89
8. What syndrome do you suspect in a 13-year-old boy with microscopic hematuria and hearing loss? 90
Differential Diagnosis 90
9. Quick review: Cover the right column in Table3-3 and attempt to diagnose the cause of the glomerulonephritis base... 90
Case3-8 91
1. What is the diagnosis? 91
2. What are the most common causes of nephrolithiasis? 91
3. How can nephrolithiasis cause renal failure? 91
4. What are the most common causes of hypercalcemia among both outpatients and inpatients? 92
5. Why is hypercalcemia secondary to hyperparathyroidism less likely to cause renal calculi formation than other causes of hy 93
Related Questions 93
6. How does urinary tract infection with bacteria such as Proteus mirabilis predispose to struvite stone formation? 93
7. How does urinary pH influence the precipitation of uric acid stones? 93
8. What is cystinuria and how does it lead to cystine stones? 93
9. How does Crohn's disease lead to an increased risk of kidney stones? 93
Case3-9 94
1. What is the most likely diagnosis? 95
2. What is the most common source of infection in pyelonephritis? 95
3. Why are pregnant women with asymptomatic bacteriuria treated more aggressively than nonpregnant women with the same... 95
4. Why are pregnant women with bacteriuria more susceptible to pyelonephritis? 95
5. What is acute interstitial nephritis and how does it differ from acute pyelonephritis? 95
6. Why are urinary tract infections in men younger than 50 often evaluated aggressively? 95
7. Why are men older than 50 predisposed to urinary tract infection? 96
Chapter 4: Fluid and Electrolytes 97
Basic concepts-renal filtration and transport processes 97
1. What forces govern the glomerular filtration rate at the level of the glomerulus? 97
2. How do angiotensin-converting enzyme inhibitors and angiotensin receptor blockers affect glomerular filtration rate? 98
3. What is meant by the term filtration fraction and how will increasing the glomerular capillary oncotic pressure (without... 98
4. What are the three layers of the glomerular ``filter´´ and how do they contribute to the process of renal filtratio... 99
5. What is the significance of the creatinine clearance and how is it measured? 99
Basic concepts-renal control of acid-base balance 101
1. Why is net renal acid excretion necessary to maintain acid-base homeostasis? 101
2. What mechanisms does the kidney use to maintain acid-base balance despite this acid load? 101
3. How do the kidneys reabsorb filtered bicarbonate? 102
4. What effect does the diuretic acetazolamide have on the acid-base balance of the body? In what clinical conditions migh... 102
5. How are bicarbonate and ammonium generated de novo by the kidney? 103
Basic concepts-renal control of extracellular fluid balance 104
1. What are the extracellular fluid compartments of the body and how do their relative sizes compare to the intracellular... 104
2. How do the kidneys regulate extracellular fluid volume? 104
3. What is the normal role of the renin-angiotensin-aldosterone system? 104
4. What is the role of the sympathetic nervous system in maintaining effective circulating volume? 104
5. How does the antidiuretic hormone regulate extracellular fluid volume? 105
Case4-1 105
1. What disease did this patient have and what is its underlying cause? 105
2. How does the renin-angiotensin-aldosterone system contribute to renovascular hypertension? 105
3. How do the kidneys regulate blood pressure independently of the renin-angiotensin-aldosterone system? 105
4. If angiotensin II promotes vasoconstriction, why doesn't the angiotensin II released during hypovolemic states reduce glom 106
5. Why should angiotensin-converting enzyme inhibitors (or angiotensin receptor blockers) be avoided 106
Renal Control of Extracellular Fluid Osmolarity 107
Case4-2 107
1. What is the most likely diagnosis and what is its cause? 107
Case4-2 continued: 107
2. What syndromes are associated with syndrome of inappropriate secretion of antidiuretic hormone ? 107
3. What is the function of antidiuretic hormone? 108
4. Why is the antidiuretic hormone release in SIADH ``inappropriate´´? 108
5. What is the result of inadequate antidiuretic hormone release? 108
6. How does hyponatremia result in the central nervous system symptoms (fatigue, anorexia, and confusion) seen in this patie... 109
7. Why must the hyponatremia be corrected slowly in this patient? 109
8. Why was it important to ask about vomiting, diarrhea, or diuretic use? 109
9. Why was it important to examine the albumin level and look for evidence of heart failure? 109
10. How would levels of plasma antidiuretic hormone, plasma osmolarity, and serum osmolarity be expected to differ between... 110
Pharmacology of Diuretics 110
1. How do diuretics work to lower extracellular fluid volume? 110
2. What percentage of the filtered sodium is reabsorbed under normal conditions (i.e., in the absence of diuretics)? 110
3. In which region of the nephron does each of the major diuretic types (carbonic anhydrase inhibitors, osmotic diuretics... 112
4. How does each major diuretic type affect the concentrations of serum electrolytes? 113
5. What are the other relatively common or important side effects of diuretics? 115
Chapter 5: Acid-Base Balance 117
Basic concepts 117
1. How is extracellular hydrogen ion concentration regulated? 117
2. How does a change in HCO3 or in Pco2 affect pH? 117
Case5-1 117
1. What is the primary acid-base disorder? 118
2. What are the mechanisms involved in metabolic acidosis due to diarrhea? 118
3. What else is considered in the differential diagnosis for non-anion gap metabolic acidosis? 119
4. Is there appropriate compensation or is there a mixed disorder in this patient? 119
5. What would it mean if, in the same patient, appropriate compensation was not present and Pco2 was instead 30mm Hg? 119
Case5-2 121
1. What is the primary acid-base disorder? 121
2. What is the differential diagnosis in this patient? 121
3. Is there appropriate compensation or is this a mixed disorder? 122
4. Does the degree of compensation allow you to draw any conclusions as to the duration of the condition? 123
Case5-3 124
1. What is the primary acid-base disorder? 124
2. What is the differential diagnosis in this patient? 124
3. What is the pathophysiology of each of the conditions in the differential diagnosis? 124
4. Is there appropriate compensation or is there a mixed disorder? 125
Case5-3 continued: 125
5. What is the most likely diagnosis? 125
6. Would administration of saline be helpful in this patient? 125
Case5-4 126
1. What is the primary acid-base disorder? 126
2. Is there appropriate compensation or is this a mixed disorder? 126
3. What is the next step in diagnosis of this disorder? 126
4. How can there be such a large anion gap with such an extremely low pH when the disturbance in HCO3- is so minimal? 127
Case5-4 continued: 127
5. What is the most likely diagnosis in this patient? 127
6. What is on the differential diagnosis for large anion gap metabolic acidosis? 129
7. What is the correct treatment? 129
Case5-5 129
1. What is the most likely diagnosis? 129
Case5-5 continued: 130
2. Now what is the most likely diagnosis? 130
3. What is the treatment for the man in questions 1 and 2? 130
Case5-6 130
1. What is the most likely diagnosis? 131
Case5-7 131
1. What is the primary acid-base disorder? 132
2. What is the differential diagnosis in this patient? 132
3. What is the most likely diagnosis in this patient? 132
4. Is there appropriate compensation or is this a mixed disorder? 132
Case5-7 continued: 132
5. How can you determine if this exacerbation is due to central hypoventilation from sedation or due to a ventilation-perf... 132
Chapter 6: Gastroenterology 134
Basic concepts 134
1. What is the major stimulus for gastrin secretion? What are the physiologic actions of gastrin in the stomach? 134
2. What are the main pancreatic enzymes and what are their functions? 134
3. What are the primary hormonal stimuli for the pancreatic exocrine secretions and how do these secretions 135
4. What are the primary stimuli for the secretion of cholecystokinin and secretin and from where are these hormones secreted? 135
5. What other digestive processes does cholecystokinin stimulate? 135
6. What is the function of the bile salts? How are they formed? 135
7. What is the enterohepatic circulation and why is it important in the digestion of fats? 135
8. What defines the foregut, midgut, and hindgut anatomically? Which main arteries provide the blood supply to each segment? 135
9. What are the anatomic layers of the gut wall? 136
Case6-1 136
1. What is the differential diagnosis? 136
Case6-1 continued: 137
2. Based on the findings in Figure6-2, what is the patient's likely diagnosis? 138
3. Discuss the pathophysiology of gastroesophageal reflux disease. 138
Case6-1 continued: 138
4. Cover the far left column of Table6-1 and attempt to name the drugs used in the treatment of GERD based on the class of ... 138
5. Which of the drugs in Table6-1 are contraindicated in a patient in whom bowel obstruction is suspected and why? 139
6. Why may the omeprazole this patient was given cause him to develop hypergastrinemia? 139
Case6-1 continued: 139
7. Which complications of gastroesophageal reflux disease could be responsible for the patient's difficulty swallowing? 139
8. Differentiate between the two types of esophageal cancers-adenocarcinoma and squamous cell carcinoma-in terms of... 139
9. Why would a patient with Sjögren's syndrome be more susceptible to esophageal pathology in gastroesophageal reflux diseas... 139
Case6-2 140
1. In pathophysiologic terms, how do you approach dysphagia? 140
Case6-2 continued: 140
2. What is the likely diagnosis in this woman? Describe the pathophysiology of this disease. 141
3. Chagas´ disease is also known to be a cause of achalasia. What is the pathologic mechanism and what organism is the... 141
4. What was the likely cause of this woman's previous episode of pneumonia? 141
5. In addition to pneumatic dilatation of the lower esophageal sphincter and surgical myotomy, injection of botulinum toxin... 141
Case6-3 143
1. What is the differential diagnosis? 143
Case6-3 continued: 143
2. If you were this man's physician, what would you have done differently in the treatment of this patient? 143
3. How are nonsteroidal anti-inflammatory drugs thought to predispose to the formation of gastric ulcers? What alternatives... 144
Case6-3 continued: 144
4. Based on the appearance of the stool, is this more likely a lower gastrointestinal bleed or an upper gastrointestinal... 144
5. What are the major complications of peptic ulcer disease? 144
Chapter 7: Hepatology 164
Basic concepts 164
1. Review the anatomy of the hepatic lobule and portal triad. In what manner do blood and bile flow through a lobule? 164
2. What is the chemical difference between conjugated and unconjugated bilirubin and how are these substances formed? 164
3. Why is unconjugated bilirubin not normally excreted in the urine? 165
4. What are the main causes of jaundice and how does each affect the type of hyperbilirubinemia observed? 165
5. Which veins feed into the portal vein? 167
6. What are the symptoms of portal hypertension? 167
7. How can Budd-Chiari syndrome arise and what can it lead to? 168
8. What are the common liver biochemical tests and what do they indicate? 168
Case7-1 169
1. What is the most common cause of upper gastrointestinal bleeding? 169
2. Why should a Mallory-Weiss tear be included in the differential diagnosis? 169
3. What is scleral icterus? 169
4. To confirm your suspicion about severe liver disease in this patient, what physical examination 169
Case7-1 continued: 169
5. What is the pathophysiology of his ascites, pedal edema, and periorbital edema? 169
6. What is the pathogenesis of the suspected cause of hematemesis in this patient? 170
Case7-1 continued: 171
7. What is the value of the following tests: hepatitis serology, serum iron, ceruloplasmin, and antimitochondrial antibodies? 171
8. Assuming he is not taking any anticoagulants, what is the most likely reason this patient developed a large hematoma... 171
9. List all the laboratory findings you would expect in a patient with liver failure. 171
10. Would you expect the ascitic fluid to be a transudate or an exudate? 172
11. How does liver cirrhosis cause the following abnormalities? 172
12. How can an acute alcohol binge cause a fatty liver to develop? 173
13. How does chronic alcohol consumption lead to a more rapid catabolism of ingested alcohol? 173
14. Why are ethanol and fomepizole used to treat methanol poisoning and ethylene glycol poisoning? 174
Case7-2 174
1. To what disease is the student referring? 175
2. What laboratory tests and further physical examination components would you like to do to strengthen your suspicion for... 175
Case7-2 continued: 175
3. How is the diagnosis of Wilson's disease made definitively? 175
4. What is the pathogenesis of Wilson's disease? 175
5. If this patient remains untreated, what neurologic manifestations may develop? 175
6. What is the treatment for Wilson's disease and how does it work? 175
Case7-3 176
1. In terms of infections, what do you include in the differential diagnosis for a patient who has recently traveled out of... 176
Case7-3 continued: 176
2. What is the diagnosis? 176
3. Why is there an elevation of both direct and indirect bilirubin? 176
4. Explain how the results of the hepatitis profile facilitate the diagnosis of an acute infection rather than a chronic one. 177
5. Should he be concerned about developing a chronic infection or hepatic cirrhosis? 177
6. How would you expect liver ``function´´ test patterns to differ between parenchymal liver disease and cholestatic... 177
7. Now let's review some characteristic features of the different hepatitis viruses. 177
8. Can hepatitis A be prevented? 177
Case7-4 179
1. What is the most likely diagnosis? 179
2. Why does physiologic jaundice develop? 179
3. Why did the physician check for hematomas on physical examination? 179
4. Why are a normal reticulocyte count and a normal complete blood count important in the diagnostic workup for this neonate? 179
5. What is the most serious complication of neonatal jaundice and how does it develop? 179
Case7-4 continued: 180
6. Are there long-term risks associated with physiologic jaundice of the newborn? 180
7. Would you expect physiologic jaundice to be exacerbated or attenuated by Gilbert's syndrome? 180
8. What is the hereditary syndrome with a more serious deficiency of uridine diphosphate 180
9. What are the hereditary forms of conjugated hyperbilirubinemia and what is the major histologic difference between them? 180
Case7-5 183
1. To what ailments are persons infected with hepatitis B susceptible? 183
Chapter 8: Endocrinology 196
Basic concepts 196
1. What is the cellular mechanism of action of the steroid hormones? 196
2. What is the cellular mechanism of action of the peptide hormones and the catecholamines? 196
3. Why is the total serum hormone level not an accurate reflection of hormone activity? 196
4. How does a hormone's binding to the same type of receptor have different effects in different cell types? 198
5. What are the four primary classes of membrane-spanning receptors to which peptide hormones bind? 198
6. How do the tyrosine kinase receptors transduce their messages? 199
7. How do the ligand-gated ion channels work? 199
8. How do the G proteins transduce their signals? 199
9. How do endocrine, paracrine, and autocrine mechanisms of cell communication differ? 200
10. Describe the concept of negative feedback. What is a feedback loop? 200
Case8-1 201
1. What is the diagnosis? 202
2. Why is the pituitary more susceptible to infarction in postpartum hemorrhage than in hemorrhagic shock unrelated to pregn... 202
3. Why is the posterior pituitary typically spared in Sheehan's syndrome? 202
4. Secretion of which pituitary hormones may be affected in this woman? 202
5. Why may hypothalamic releasing hormone secretion increase because of an infarction of the anterior pituitary? 203
6. Why wouldn't hypothalamic dopamine secretion be elevated from an anterior pituitary infarction? 205
Case8-2 207
1. What is the diagnosis? 207
2. What are the normal physiologic functions of prolactin preceding, during, and following pregnancy? 207
3. Why does this patient have galactorrhea, whereas pregnant women with similar levels of serum prolactin 207
4. Hyperprolactinemia can also occur in men. What symptoms might be expected in men? 207
5. How does elevated prolactin prevent pregnancy (i.e., what is the mechanism of infertility and amenorrhea in this patient)? 209
6. Why is asking about a history of schizophrenia and use of antipsychotic medications a relevant question in the diagnostic.. 209
7. What is the mechanistic basis for using bromocriptine (used to treat Parkinson's disease) in the treatment of a prolactin... 209
8. How can head trauma with a severed pituitary stalk cause a similar increase in prolactin (assuming the anterior pituitary... 209
9. Why does hypothyroidism need to be considered in the evaluation of hyperprolactinemia? 209
Case8-3 210
1. What is the diagnosis? 210
2. Why is hyperglycemia commonly associated with this disease? 210
3. What are the normal physiologic functions of growth hormone and how is its secretion regulated? 210
4. Given the normal physiology of growth hormone, how can we explain this patient's presentation? 211
5. Why is octreotide, a somatostatin analog, useful in the treatment of acromegaly? 211
6. If this patient developed a growth hormone-secreting tumor in his early teens, how might the clinical manifestations... 212
7. What growth abnormality results from deficient secretion of growth hormone during the growing years? 212
Case8-4 212
1. What are the general causes of the hormonal abnormality most likely present in this woman? 212
Case8-4 continued: 213
2. What is the cause of the hypercortisolism in this patient? 213
3. Why are the results of a dexamethasone suppression test read at a specific time of day? 213
4. Why has hirsutism developed in this woman? 213
5. Why isn't hyperaldosteronism typically seen in Cushing disease? 214
6. What morphologic feature of the adrenal glands would you expect to see in this woman? 214
7. How is hypercortisolism contributing to hyperglycemia in this patient? 214
8. How is hypercortisolism contributing to hypertension and hypokalemia in this patient? 214
9. Why doesn't cortisol have mineralocorticoid actions in the normal physiologic setting? 214
10. What would an x-ray study of her bones likely reveal? 215
11. What are the three layers of the adrenal cortex, and which one is responsible for the excess production of cortisol in... 215
12. One treatment for Cushing disease is to remove both adrenal glands (bilateral adrenalectomy). What might happen to the... 215
Case8-5 216
1. What do you suspect at this point? 216
2. What are some general causes of primary adrenal insufficiency (Addison's disease)? 217
Case8-5 continued: 217
3. What was the cause of adrenal insufficiency in this patient? 217
Related Questions 217
4. What adrenal disease should be suspected in a young patient with bacterial meningitis due to Neisseria meningitidis who... 217
5. How would we expect plasma aldosterone levels to be affected in a patient with secondary 217
Case8-6 218
1. What do you suspect at this point? 218
2. Why does hypertension often worsen after starting a beta blocker in patients with pheochromocytoma? 218
Case8-6 continued: 218
3. What is the likely diagnosis? 218
4. How can the administration of clonidine be used to differentiate pheochromocytoma from a ``high-stress state´´? 219
5. What is the ``rule of 10s´´ for pheochromocytomas? 219
6. What malignant tumor that most often occurs in children under 5years of age also shows increased 219
Case8-6 continued: 219
7. What type of receptors do norepinephrine and epinephrine bind to on the heart to increase the rate and force of cardiac co.. 219
8. Why was this patient given propranolol, a nonselective beta blocker, if it can cause a hypertensive crisis? 220
9. On a related note, what class of antihypertensive agent, if given prior to epinephrine, would make it so that epinephrine... 220
Case8-6 continued: 220
10. Why was phenoxybenzamine given prior to surgery? 220
Case8-7 221
1. What do you suspect at this point? 221
Case8-7 continued: 221
2. What is the diagnosis? 221
3. Why has this woman experienced weight loss? 223
4. What are the two thyroid hormones and which is more potent? 224
5. How are the thyroid hormones synthesized? 224
6. Assuming the plasma levels of catecholamines are normal in this patient, what explains the tachycardia, tremors, palpitati 224
7. What is the difference between primary, secondary, and tertiary hyperthyroidism and which does this patient have? 224
8. Based on your suspected diagnosis, what additional laboratory and physical findings might you expect? 225
Case8-7 continued: 225
9. What is the pathophysiology of this complication of Graves´ disease? 225
10. What would a thyroid iodide-131 uptake scan likely reveal in this patient? 225
11. What therapeutic options are available to this patient? 227
12. Why might a physician prescribe propylthiouracil or methimazole for this woman? How do these drugs work? 227
13. What is thyroid storm? Why is propylthiouracil used for this condition instead of methimazole? 228
14. Why is iodide therapy generally initiated 2weeks prior to thyroidectomy in hyperthyroid patients? 228
Some Differential Diagnosis and Physiology Concepts 228
15. How would a patient with hyperthyroidism secondary to de Quervain's thyroiditis (subacute granulomatous thyroiditis) typi 228
16. How can a teratoma produce hyperthyroidism? 228
Case8-8 229
1. What do you suspect at this point? 229
Case8-8 continued: 229
2. Why was asking about lithium use relevant in the diagnostic workup of this patient? 229
Chapter 9: Male and Female Reproductive Systems 252
Basic concepts 252
1. What is the normal duration of the menstrual cycle? What are the two ovarian phases, and which occurs first? 252
2. Which phase of the ovarian cycle is generally responsible for the cycle being longer or shorter? 252
3. During the follicular phase, what hormonal changes occur in the pituitary and the ovary?: 252
4. What occurs in the uterus during the follicular phase? 253
5. What happens to cause ovulation at the end of the follicular phase? 253
6. What happens in the ovary and endometrium during the luteal phase? 254
7. In women with amenorrhea (absence of menses), why does bleeding after the cessation of a brief course of progesterone indi 255
8. How does fertilization prevent degeneration of the corpus luteum? 255
9. How does the corpus luteum function in the maintenance of pregnancy? 255
10. Describe the hormones associated with normal testicular descent. 255
11. What is the function of the SRY gene? 255
12. What is hermaphroditism? 255
13. What is 5α-reductase deficiency? 256
Case9-1 257
1. What important information should you find out before prescribing hormonal contraceptives? 257
Case9-1 continued: 257
2. What are you concerned about in this woman's history in regard to hormonal contraception? 257
3. What are the absolute contraindications to using estrogen-containing contraceptives? 257
4. What do oral contraceptives typically consist of and what is their mechanism of action? 257
5. How can menstrual cycles be made regular by hormonal contraceptives? 258
6. Your patient asks if taking hormonal contraceptives will increase her risk for endometrial cancer. How does taking oral co 258
7. How do ovarian cysts form and how do oral contraceptives reduce their occurrence? 258
8. Why may the drugs phenytoin, phenobarbital, and rifampin make oral contraceptives less effective at preventing pregnancy? 258
Case9-2 259
1. What is the differential diagnosis for secondary amenorrhea? 259
Chapter 10: Oncology 291
Basic concepts-cell biology of cancer 291
1. What is the difference between an oncogene and a tumor suppressor gene? 291
2. Which type of mutation is more commonly involved in familial cancer syndromes? 291
3. How does Knudson's ``two-hit hypothesis´´ relate to tumor suppressor genes? 291
4. What are the phases of the cell cycle? Where is the restriction point? 292
5. What is the function of checkpoints and cell cycle arrests? 292
6. What is the importance of p53? 293
7. What is the function of the retinoblastoma protein? 293
8. When can a solid tumor be detected clinically? 293
9. What is the importance of angiogenesis in solid tumor growth? 293
10. What is dysplasia? What is anaplasia? 294
Basic concepts-cancer epidemiology 294
11. What are three leading causes of death in the United States? 294
12. Aside from skin cancer, which cancers have the highest incidence in men and in women? Which cancers are the leading cause 294
Basic concepts-cancer classification 294
13. What is the difference between ``grade´´ and ``stage´´ of a neoplasm? 294
14. What is the difference between a benign tumor and a malignant tumor? 295
15. How are cancers named according to the cell type they originate from? 295
Case10-1 295
1. What is dyspepsia? 296
2. What is the differential diagnosis for dyspepsia? 296
Chapter 11: Genetic and Metabolic Disease 329
Basic Concepts 329
1. What is an enzymopathy and how does it result in clinical symptoms? 329
2. What is the typical pattern of inheritance observed in enzymopathies? 330
3. Explain why the pathologic consequences of X-linked enzymopathies are manifested almost exclusively in males. 330
4. What is the process of lyonization and how may it cause the manifestation of X-linked diseases in females? 330
5. Why do some diseases show an autosomal dominant pattern of inheritance? Why do the genetic diseases of connective tissue u 331
6. What is the general relationship between the function of a protein and its pattern of inheritance? 331
7. What are the following molecular biology diagnostic methods used for? Explain briefly how they work. 331
Case11-1 332
1. What is the most likely diagnosis in this baby and how is it inherited? 333
2. What is the major defect and underlying pathophysiology of this disorder? 333
3. How is phenylketonuria treated? 334
4. Given the fact that phenylketonuria is a relatively rare condition (prevalence rates range from 1 in 2600 to 1 in 200,000 334
5. If the parents have a female child with this disease, why is it crucial to advise the child about the risks to her baby if 334
Related Question 334
6. Why is screening for congenital hypothyroidism (cretinism), congenital adrenal hyperplasia, and galactosemia also routinel 334
Case11-2 335
1. If the parents decide to have another child, what is the probability of that child having cystic fibrosis? 335
2. If the parents want to have another child, what kind of genetic screening methods are available for them to consider? 335
3. Despite having mutations in the same gene, why do patients with cystic fibrosis exhibit significant variability in disease 335
4. Assuming a cystic fibrosis prevalence rate of 1 in 2500, what is the carrier frequency for this disease? 336
Case11-3 336
1. What two diagnoses are top considerations in the differential diagnosis at this point? 337
Case11-3 continued: 337
2. Now what is the most likely diagnosis? 337
3. What is the pathogenesis of Tay-Sachs disease? 337
4. Which is the most common sphingolipidosis? 337
5. What are the mucopolysaccharidoses? 338
6. Quick review: Cover the three columns on the right side of Table11-3 and attempt to describe the enzyme deficiency, accu 338
Case11-4 340
1. What is the most likely diagnosis? 340
2. What is the normal function of the purine ``salvage´´ pathway? 340
3. How do defects in the purine salvage pathway cause hyperuricemia? 341
4. What was the orange ``sand´´ in his diapers observed by his parents? 341
5. Why do boys with Lesch-Nyhan syndrome typically present with renal dysfunction? 342
6. How might this patient be managed pharmacologically? 342
Case11-5 342
1. What is the diagnosis? 343
2. What type of enzymatic deficiency is present in all types of glycogen storage diseases? 343
3. How is glycogen normally synthesized and degraded in the liver? 343
4. What is the function of glucose-6-phosphatase? 344
5. Why is hepatomegaly seen on examination? 344
6. What is the explanation for his severe fasting hypoglycemia and lactic acidosis? 344
7. Why is this patient susceptible to hypertriglyceridemia? 344
8. What causes the hyperuricemia in this patient? 345
9. Quick review: Cover the columns on the right side of Table11-4 and explain how these diseases affect the activity of the 345
Related Questions 346
10. Why does a deficiency of muscle glycogen phosphorylase (seen in type V glycogen storage disease, or McArdle disease) not 346
11. Review the high-yield glycogen storage diseases. 346
Case11-6 348
1. What is the most likely diagnosis? 348
2. What are the three length classifications of fatty acids? 348
3. What are the reasons for the clinical and laboratory findings exhibited by this patient? 348
4. How should this child be treated? 349
5. How are the pathways/cycles listed in Table11-6 affected by medium-chain fatty acyl-CoA dehydrogenase deficiency? 349
Case11-7 350
1. What are the porphyrias? 350
2. What are the types of porphyria? 350
3. Why may cigarette smoking or anticonvulsants such as phenytoin and phenobarbital trigger acute porphyrias? 351
4. Why is hemin or glucose given to patients with acute porphyrias? 351
Related Questions 351
5. How does lead poisoning affect heme synthesis? 351
6. What are the sideroblastic anemias? 351
Case11-8 352
1. What is the diagnosis? 352
2. Why does galactose-1-phosphate uridyltransferase deficiency manifest with jaundice? 353
3. Why do infants with galactosemia tend to develop cataracts if left untreated? 353
4. What is the treatment for classical galactosemia? 353
Case11-9 354
1. What is the diagnosis? 354
2. Why does this patient exhibit multiple pathologic presentations in the ocular, skeletal, and cardiovascular systems? 354
3. What pathologic condition would you suspect in a patient with Marfan syndrome who complains about ``the worst headache of 355
4. What are the major cause(s) of death in this disorder? 355
Related Questions 355
5. What hereditary skeletal disease predisposes to bone fractures from minor stress? What structural protein defect underlies 355
6. What is the genetic defect in Ehlers-Danlos syndrome? 355
Case11-10 356
1. What is the most likely diagnosis? 356
2. What is the etiology of this disease? 356
3. How does the cystic fibrosis transmembrane regulator (CFTR) mutation lead to disease? 356
4. Why does this child develop respiratory infections so easily? 357
5. Why might this child be susceptible to developing pancreatitis as he grows older? 357
6. What is bronchiectasis and why does it commonly develop in cystic fibrosis? 357
7. Why is N-acetylcysteine used as a treatment for cystic fibrosis? 357
Related Questions 358
8. Why does Kartagener's syndrome produce clinical manifestations similar to those of cystic fibrosis? 358
Case11-11 359
1. What is your diagnosis based on this patient's family history and fasting lipid profile? 359
2. Based on his clinical presentations, is this patient likely to be heterozygous or homozygous for this deficiency? 359
3. What is the primary mechanism by which mutations in the low-density lipoprotein receptor impair low-density lipoprotein up 360
4. What is the pathologic consequence of elevated plasma low-density lipoprotein? 360
5. How might this patient be managed medically and pharmacologically to decrease his risk of future cardiovascular complicati 360
Related Question 361
6. How do HMG-CoA reductase inhibitors specifically reduce serum low-density lipoprotein levels? 361
Case11-12 361
1. What is the most likely diagnosis? 361
2. What is the pathogenesis of this disorder? 362
3. Southern blot analysis indicates that his mother has 90 CGG trinucleotide repeats, whereas he has 350 CGG repeats. What ac 362
Related Question 363
4. What other relatively common inherited disorder exhibits genetic anticipation? 363
Case11-13 363
1. What is the most likely diagnosis? 364
2. What is a robertsonian translocation and how does a robertsonian translocation in a parent result in Down syndrome in the 364
3. What is the mechanism that gives rise to mosaic Down syndrome, whereby only select tissues express the trisomy 21? 364
4. If the child were to fail to initiate proper feeding, what abnormality should be suspected? 364
5. What causes most of the deaths in infancy and in childhood in Down syndrome? 365
6. Which cancer types are associated with Down syndrome? 365
7. What characteristic neuropathologic changes are seen in brains of older people (i.e., >40years of age) with Down syndrome? 365
Related Questions 365
8. What other two autosomal trisomies can sometimes produce live-born infants? 365
9. What two diseases result from microdeletion of the same section of chromosome 15? 365
10. Why is the parental source of the chromosome significant in the aforementioned microdeletion syndromes? 366
Case11-14 367
1. What is the (noncardiac) diagnosis in this patient? 367
2. What is the reason for his elevated urinary urea nitrogen excretion? 367
3. What is the reason for the low serum albumin level? 368
4. Would you expect this patient to have an obviously emaciated appearance and muscle wasting? 368
5. What are some potential dangers if this condition is left treated? 368
6. Compare and contrast marasmus and kwashiorkor 368
Case11-15 369
1. What is the likely diagnosis in this patient? 369
2. What are the symptoms of vitamin A toxicity? 370
3. What are the symptoms associated with vitamin A deficiency? 370
Chapter 12: Anemias 371
Basic concepts 371
1. What is anemia and how is it defined? 371
2. What are the three pathophysiologic mechanisms resulting in anemia? 372
2. What are reticulocytes? What is a normal reticulocyte count? 372
3. What are the hemolytic anemias and how are they typically classified? 372
4. Regarding hemolytic anemia, what is the difference between intravascular and extravascular hemolysis? 373
5. Why is there a greater degree of hemoglobinemia and hemoglobinuria in intravascular hemolysis than in extravascular hemoly 373
6. How are microcytic, macrocytic, and normocytic anemias defined? 373
7. What is the usual cause of anemia in end-stage renal failure? 374
Case12-1 374
1. What is the differential diagnosis for normocytic anemia? 374
Case12-1 continued: 374
2. What is the expected diagnosis now and what is the confirmatory test? 375
Chapter 13: Bleeding Disorders 404
Basic Concepts 404
1. Differentiate between the processes of primary and secondary hemostasis. 404
2. What molecule is responsible for the binding of platelets to collagen? 404
3. What constitutes the extrinsic, intrinsic, and common pathways in the coagulation cascade that forms the fibrin clot (seco 404
4. What information can be provided by measuring the prothrombin time and activated partial thromboplastin time? 405
5. What is the bleeding time? What is its clinical significance? 405
6. What is the mechanism of action of the following drugs? How do they affect the times just discussed? 405
7. What is the mechanism of action of tissue plasminogen activator? 407
Case13-1 408
1. What is the most likely diagnosis? 408
2. What is the cause of this disorder and how is it inherited? 408
3. Which measure of coagulation will be abnormal in hemophilia A? 409
4. What is the mainstay of medical treatment for this disease? 409
Case13-2 409
1. What is the most likely diagnosis? 409
2. What is the pathogenesis of disseminated intravascular coagulation? 410
3. What are schistocytes and why do they form in disseminated intravascular coagulation? 410
4. How does the pathophysiology of thrombotic thrombocytopenic purpura differ from that of disseminated intravascular coagula 410
5. What is the pentad of thrombotic thrombocytopenic purpura and how does this disorder differ from hemolytic uremic syndrome 411
6. Quick review: Cover the two columns at the right in Table13-3 and try to differentiate thrombotic thrombocytopenic purpu 411
Case13-3 412
1. What is the most likely diagnosis? 412
2. What is the normal function of von Willebrand factor and what is the pathogenesis of this man's disease? 412
3. Why may someone with von Willebrand disease be mistakenly diagnosed with hemophilia A? 413
4. What is the mechanism of action whereby administration of desmopressin acetate might help this man's symptoms? 413
Differential Diagnosis 413
5. If this man had normal levels of functional von Willebrand factor, and platelet function studies revealed a defect in plat 413
6. If platelet function studies demonstrated platelets capable of adhering to collagen but unable to aggregate with other pla 413
7. Given the previously mentioned function of the GPIIb-IIIa receptor, why are drugs such as abciximab (Integrilin) given to 414
8. Why might you suspect an abnormal bleeding time in this man if he suffered from diabetic nephropathy and osteoarthritis fo 414
9. Quick review: Cover the two columns on the right side of Table13-4 and try to describe the mechanisms of action for the 414
Case13-4 415
1. What is the most likely diagnosis in this woman? 415
2. What is the etiology of immune thrombocytopenic purpura? 415
3. Would a bone marrow biopsy in this woman likely reveal increased or decreased numbers of megakaryocytes and why? 415
4. What anticoagulant is well known for causing thrombocytopenia? 415
5. Why is the absence of splenomegaly important and therefore helpful in determining the cause of thrombocytopenia? 415
6. Can pregnant women with this disease affect the platelet count of their fetuses? 416
7. What is the treatment strategy for this disease? 416
Case13-5 416
1. What is the diagnosis? 416
2. From what site do deep venous thromboses, which give rise to pulmonary embolisms, typically arise? 417
3. What is Virchow's triad and how does this relate to this patient? 417
4. How do deficiencies of proteins C and S and antithrombin III predispose to deep venous thrombosis? 417
5. What is antiphospholipid syndrome? 417
Chapter 14: Hematologic Malignancies 418
Basic Concepts 418
1. What are the two principal lineages along which leukocytes differentiate? 418
2. What categories of hematologic malignancy arise from the lymphoid lineage? 419
3. What is the general distinction between lymphoma and leukemia? 419
4. What is the distinction between small cell lymphocytic lymphoma (SLL) and (B-cell) chronic lymphocytic leukemia (B-CLL)? 419
5. What categories of hematologic neoplasms arise from the myeloid lineage? 419
6. What is the distinctive feature of acute myelogenous leukemia on bone marrow biopsy? 419
7. What is the distinctive feature of myelodysplastic syndromes? 420
8. What is the distinct feature of the myeloproliferative disorders? 420
9. What are the histiocytoses? 420
10. What is the relationship between myelofibrosis and the myeloproliferative diseases? 420
11. How does the leukocyte alkaline phosphatase level help differentiate reactive leukocytosis from a true leukemia? 421
12. Quick review! Cover the right column in Table14-1 and attempt to list the cell type and pertinent high-yield facts reg 421
13. Quick review! Cover the right column in Table14-2 and attempt to define the hematologic terms in the left column. 421
14. What are the genetic alterations in the following non-Hodgkin's lymphomas? 422
15. What are some characteristics of the other non-Hodgkin's lymphomas? 423
16. Quick review with high-yield word associations: Cover the right column of Table 14-5 and attempt to list the associated d 423
Case14-1 423
1. What is the differential diagnosis for back pain in an older patient? 424
Case14-1 continued: 424
2. What is the expected diagnosis? 424
3. What blood abnormalities often present with multiple myeloma and why? 424
Case14-1 continued: 425
4. What cell type abnormally proliferates in multiple myeloma? 425
5. What is the association between Bence Jones proteinuria and the previously mentioned monoclonal gammopathy? 425
6. What is amyloidosis and why is this man at risk for developing it? 426
7. Explain why this man is at an increased risk for infection even though plasma levels of immunoglobulins are abnormally ele 426
8. This patient's anemia can be characterized as myelophthisis. Why? 426
9. What is the characteristic finding on a peripheral blood smear, as shown in Figure14-3, and why does this occur? 426
10. How might the presence of elevated serum κ light chains and amyloidosis affect kidney function in this patient? 426
11. What is the pharmacologic basis for giving this patient allopurinol prior to and during chemotherapy? 426
Differential Diagnosis 427
12. If workup reveals an IgM monoclonal gammopathy rather than an IgG or IgA gammopathy, what disease might you suspect? 427
Case14-2 427
1. Her parents are worried she might have cancer. What is the most common cancer in children? 427
Chapter 15: Immunology 439
Basic Concepts 439
1. Outline hematopoiesis, beginning with a pluripotent stem cell. 439
2. What are the major primary and secondary organs that make up the human lymphoid system? 439
3. What is the function of innate immune system? 439
4. What is adaptive/acquired immunity? 440
5. What are the basic characteristics of cell-mediated and humoral immunity? 440
6. Describe the difference between class I and class II major histocompatibility complex molecules. 441
7. How do antibodies eliminate extracellular pathogens? 442
8. By what process can antibodies catalyze the elimination of intracellular pathogens? 442
9. What are the five classes (isotypes) of immunoglobulins? Describe their respective distributions in the body. 442
10. Most humans can produce 106 to 109 unique immunoglobulin (Ig) molecules. However, the number of immunoglobulin genes is o 442
11. What are complement proteins and how do they function in an immune response? 444
12. Describe the ramifications of the most common complement protein deficiencies. 444
13. Which complement components and cytokines are required for neutrophil chemotaxis? 444
14. As a review, list the effector functions of the major leukocyte classes. 445
15. List the functions of the major cytokines secreted by various classes of immune cells. 449
16. List the major cell surface markers used to identify various classes of immune cells. 449
Case15-1 449
1. What is the differential diagnosis for this presentation? 449
Case15-1 continued: 449
2. What is the most likely diagnosis? 449
3. Can free penicillin cause anaphylaxis? 450
4. What type of hypersensitivity is anaphylaxis? What is its immunopathogenesis? 450
5. What is the pathophysiologic explanation for the wheezing and diarrhea that developed? 451
6. How does anaphylaxis result in the urticaria observed in this patient? 451
7. Why did the child NOT have a reaction to amoxicillin when it was first administered for his previous ear infection? 451
8. What clinical testing can be performed to confirm that this immediate hypersensitivity reaction was caused by amoxicillin? 451
9. Why was the child immediately given epinephrine? 451
10. Why was the child given diphenhydramine and methylprednisolone? 451
Related Questions 452
11. What was the motivation for developing the second-generation H1 receptor antagonists such as fexofenadine (Allegra) and l 452
12. List the common classes of drugs that are used to treat type I hypersensitivity disorders and their general mode of actio 452
Case15-2 452
1. What is the differential diagnosis for this presentation? 452
2. What additional tests should be ordered to further analyze the anemia and jaundice? 452
Case15-2 continued: 454
3. What is the most likely diagnosis? 454
4. What is the significance of a positive direct Coombs´ test and how does it support the diagnosis considered in this pati 454
5. How do medications such as penicillin cause autoimmune hemolytic anemia? 454
6. What type of hypersensitivity does autoimmune hemolytic anemia represent? 454
7. What is the mechanism by which hemolysis occurs in drug-induced autoimmune hemolytic anemia? 454
Related Question 455
8. If this patient had had a Mycoplasma pneumoniae infection rather than a pneumococcal infection, what type of autoimmune he 455
Case15-3 455
1. What is the differential diagnosis for this boy's presentation? 455
Case15-3 continued: 455
2. What is the most likely diagnosis? 455
3. What is the pathogenesis of serum sickness? 456
4. What is the reason for the delay from the serum administration to the onset of symptoms? 456
5. What is the significance of decreased serum levels of complement? 456
6. What caused the hives and facial swelling in the patient? 456
7. What is the significance of red blood cells and protein in the urine? 456
8. What is the cause of this boy's joint pain? 457
9. Does a type III hypersensitivity reaction require previous exposure (sensitization) to antigen to occur? 457
10. Why has the increasing use of antibody therapy NOT led to a sharp increase in the incidence of immune complex-mediated 457
Case15-4 457
1. What is the differential diagnosis for this woman's rash? 457
2. How can allergic type and irritant type contact dermatitis be differentiated? 458
Case15-4 continued: 458
3. What is the diagnosis? 458
4. What is the causative agent and the mechanism by which it induced an immune response in the patient? 458
5. Why did the patient have lesions in areas other than on her arms and upper chest (where she applied the spray)? 458
6. What is the treatment for allergic type contact dermatitis? 459
7. Why is it important for the patient to avoid the use of benzocaine in the future? 459
Case15-5 459
1. What is the differential diagnosis? 459
Case15-5 continued: 459
2. What is the most likely diagnosis in this infant, given the fact that specialized testing revealed defects in both cellula 461
3. Why might a bone marrow transplant from an appropriate donor cure this boy? 461
4. What is the significance of the marked lymphopenia and complete lack of CD3 cells? 461
5. Why are B-cell defects not evident in many babies when they are first diagnosed with severe combined immunodeficiency? 461
6. Mutations in the γ subunit of the IL-2 receptor are found in the most common, X-linked form of severe combined immun 461
Related Questions: B-Cell Disorders 462
7. Which primary immunodeficiency should be suspected in a child with normal cell-mediated immunity but almost complete absen 462
8. Which B-cell disorder is associated with anaphylactic transfusion reactions? 462
Case15-6 463
1. What is the differential diagnosis for this boy's apparent immunodeficiency? 463
Case15-6 continued: 463
2. What is the diagnosis? 463
3. What are the two mechanisms that a macrophage can employ to kill bacteria following phagocytosis? 464
4. What is the genetic basis of chronic granulomatous disease? 464
5. What are the contents of a granuloma and why is it formed? 464
6. Patients with chronic granulomatous disease often experience recurrent staphylococcal infections, but streptococcal infect 464
7. What is the mechanism of action of amphotericin B? 464
8. How does liposomal amphotericin B differ from amphotericin B? 465
Related Questions: Phagocyte Disorders 465
9. How does Chédiak-Higashi syndrome differ from chronic granulomatous disease? 465
10. Compare and contrast monocytes and macrophages with respect to origin, location, lifespan, and function. 466
Case15-7 469
1. What is the likely diagnosis in this infant and what is the pathophysiology of her disorder? 469
2. Joseph Heller might ask, ``What tried and true medical school mnemonic can be used to remember the classic manifestations 469
3. To what type of infections might this child be vulnerable, given that thymic development is abnormal? 469
4. What might a lymph node biopsy in this infant reveal? 470
Related Question 470
5. What does the process of ``thymic education´´ involve? 470
Case15-8 470
1. What is the differential diagnosis? 470
Chapter 16: Psychology 486
Case16-1 486
1. What is the most likely diagnosis? 486
2. According to the Diagnostic and Statistical Manual of Mental Disorders (DSM-IV), what criteria must be met in order to mak 486
3. What is the diagnosis if this man had suffered these symptoms for only the past 3months rather than for 8months (with a ne 487
4. What is the likely diagnosis if this man presented with these symptoms and later developed depressive, manic, or mixed fea 487
5. What would your diagnosis be if this man had symptoms of schizophrenia following a severe stressor and these symptoms reso 487
6. What are the four primary types of schizophrenia? 488
7. What neurotransmitter abnormality is thought to play the primary role in this man's disorder? 488
8. What are the differences between the positive and negative symptoms experienced by schizophrenics? 488
9. What is the relationship between schizophrenia and suicide? 488
10. When initiating therapy for patients like this college student, it is important to keep potential side effects in mind an 489
11. What is the correct treatment for the previously mentioned extrapyramidal side effects? 489
12. How might this man develop the following symptoms if he is being treated with low-potency typical antipsychotics such as 489
13. Perhaps the most feared complication of antipsychotics is an idiosyncratic reaction characterized by severe muscle rigidi 490
14. How do typical and atypical antipsychotics differ with respect to their mode of action and to their effect on positive an 490
15. Which one of the four major dopamine pathways of the brain is responsible for the following symptoms in schizophrenia? 490
16. Recent studies have suggested that there is no large difference in effectiveness and tolerability between the typical and 491
17. Why is clozapine recommended for use only in schizophrenics whose symptoms are refractory to treatment with other antipsy 491
Case16-2 492
1. What is the diagnosis? 492
2. Was the previous depressive episode required to make the diagnosis of bipolar in ``Roller-coaster´´? 493
3. The physician prescribes lithium and informs ``Roller-coaster´´ that he needs to have his blood levels of lithium 493
4. After taking lithium for an extended period of time, ``Roller-coaster´´ develops polyuria and polydipsia. The urin 493
5. True or false: Treatment of this lithium-induced nephrogenic diabetes insipidus with loop diuretics may be effective in de 493
6. ``Roller-coaster´´ also mentions that he has become rather depressed after being on the lithium for a while, is havi 493
7. Because ``Roller-coaster´´ is not tolerating lithium well, his physician decides to substitute a drug that is effe 493
8. At his next visit, ``Roller-coaster's´´ symptoms seem to be well controlled with valproic acid, but his liver enzy 494
9. Why should valproic acid be used with extreme caution in patients also taking phenobarbital? 494
10. In someone with a seizure disorder that is well controlled with phenytoin, why may the addition of carbamazepine cause se 494
Case16-3 494
1. What is the most likely diagnosis? 495
2. According to the DSM-IV, what criteria must be met in order to make the diagnosis of major depressive disorder? 495
3. What does the monoamine deficiency theory propose with respect to the etiology of depression? 495
4. Why does hypothyroidism have to be ruled out in this patient? 495
5. What pharmacologic therapies are available to treat depression? 495
6. On review of systems he expresses concern about a history of premature ejaculation. What class of antidepressant may help 496
7. Why have the selective serotonin reuptake inhibitors become first-line treatments for depression over the tricyclic antide 496
8. Why might you want to avoid administering selective serotonin reuptake inhibitors and other antidepressants to this patien 496
9. What class of antidepressant was this man likely started on if he experienced symptoms of dry mouth, blurred vision, const 496
10. Assume that this patient responded well to some form of antidepressant therapy but then presented to the emergency room 3 496
11. If this man is addicted to red wine with cheese, what class of antidepressant should be avoided and why? 496
12. How do monoamine oxidase inhibitors work and what are the two classes of monoamine oxidase inhibitors? 497
13. What is the main danger of prescribing both a selective serotonin reuptake inhibitor and a monoamine oxidase inhibitor? 497
14. If this patient is suffering from depression and is additionally a smoker who is trying to quit, which drug might be effe 497
15. If this man experienced much milder symptoms of depression for longer than 2years, what would be his probable diagnosis? 497
16. How might your diagnosis change if this man had been divorced 2months ago and his symptoms of depression were milder? 497
17. If this man's wife died 1year ago and he was still experiencing these symptoms, what would his probable diagnosis be? 498
Case16-4 498
1. Instead of speaking to his deceased brother, which he believes would be unacceptable, he begins to keep a diary, which he 498
2. Why does this young man begin to wear his brother's clothing? What term is used to describe this type of activity? 499
3. What term is used to describe him taking out his frustrations on his younger brother? 499
4. What are the categories of defense mechanisms and what are some examples of each type? 499
Case16-5 499
1. What are the five stages of grief this man will likely experience? 499
2. What term is used to describe alleviating his frustration by abusing his wife? 499
3. What defense mechanism would he be employing if he ignored the doctor's visit and went on with his life without acknowledg 499
4. What term would be used to describe his behavior if while hospitalized he begins crying for his mother and demanding that 500
5. Cover the left column of Table 16-1 and attempt to name the defense mechanisms described in the middle column. 500
Case16-6 501
1. What are the considerations in the differential diagnosis? 501
2. What is this patient's likely diagnosis? 501
3. Using the DSM-IV criteria, under what axis would ADHD be listed? 502
4. What class of drugs is the primary treatment for ADHD? 502
5. What psychiatric symptoms might be evident in an individual following an overdose of amphetamines? 502
Case16-7 502
1. What is the differential diagnosis? 503
2. What are the criteria for Asperger's syndrome? 503
3. What if the patient also repeated odd phrases and was unable to successfully engage in a conversation with his peers? 503
4. How does autistic disorder differ from Asperger's syndrome? 503
5. What is known about the etiology of autism spectrum disorders? 503
6. What if the patient was not doing well academically and these behaviors and social impairments were manifested only at sch 503
7. What should the workup include? 503
8. What would be the diagnosis if the patient scored 70 or below on IQ testing? 503
Case16-8 504
1. If the psychiatrist explains this boy's developmental maturation in terms of psychosexual development, what psychologist i 504
2. Freud also discusses the id, ego, and superego. How do these concepts relate to this boy's problem? 504
3. If the psychiatrist discusses the boy's development in terms of stages of cognitive development, what psychologist is he r 504
4. If the psychiatrist explains this boy's development in terms of development of the ego, what psychologist is he referring 505
Case16-9 506
1. What is the most likely diagnosis? 506
2. What is the most likely etiology? 506
3. What concern does this man's alcohol abuse pose to the medicine team? 506
4. What are the expected symptoms of withdrawal and how are they managed? 506
5. What is the mechanism by which the administration of benzodiazepines is able to control the delirium tremens? 507
6. Another class of drugs act as agonists of the GABA receptor at a different site. What is the name of this group of drugs a 507
7. How are substance abuse and dependence differentiated? 507
Case16-10 508
1. Abuse involving what class of drugs should be expected in this man? 508
2. Would you be surprised if this man's electrocardiogram revealed myocardial ischemia? 508
3. What is the correct treatment for cocaine-induced coronary vasospasm? 508
4. Why does this man have blood around his nostril? 508
5. What are typical symptoms experienced by a person who is withdrawing from use of cocaine? 508
6. A favorite on boards is to provide an emergency department presentation of somebody with a drug overdose and ask you to de 508
Case16-11 510
1. Would this woman's current state be best described as dementia or delirium? 510
2. What is the most likely cause of this woman's delirium? 510
3. Distinguish dementia from delirium regarding onset, course, level of consciousness, and presence of delusions and hallucin 510
Case16-12 511
1. What is the most likely diagnosis in this woman? 511
2. If this woman subsequently developed a fear of leaving the house, what term should be used to describe this ``phobia&rdqu 511
3. What are the considerations in the differential diagnosis for panic disorder? 511
4. Why might the emergency department intern wish to check this woman's blood levels of thyroid hormone and urinary vanillylm 512
5. How can this woman's panic disorder be treated? 512
Case16-13 512
1. What is the most likely diagnosis in this girl? 512
2. What are the requirements needed to make this diagnosis? 512
3. Is this condition very disabling to patients and how is it best managed? 513
Case16-14 513
1. What conditions are considered in the differential diagnosis for this patient? 513
Case16-14 continued: 513
2. What is this patient's likely diagnosis? 513
3. Why does anorexia lead to amenorrhea? 513
4. What other conditions are associated with anorexia nervosa? 513
5. Compare the diagnoses of anorexia nervosa and bulimia nervosa. 514
6. How should this patient be managed? 514
Case16-15 515
1. What type of personality disorder is this man likely suffering from? 515
Case16-16 515
Chapter 17: Neurology 519
Basic Concepts 519
1. What is a motor unit? Will most α motor neurons innervate a few or many muscle fibers in a large muscle such as the glut 519
2. How do upper motor neurons differ from lower motor neurons? 520
3. What is the primary function of the cerebellum in movement? 521
4. Why do cerebellar lesions classically produce ipsilateral symptoms? 521
5. What are the two ascending sensory pathways and what information does each convey? 521
6. What are the two anatomic divisions of the dorsal columns, and from which anatomic structures do these respective division 521
7. At what neuroanatomic locations do projections in the corticospinal tract, dorsal columns, and anterolateral system (spino 521
8. Because you know where the major motor and sensory pathways cross over, identify and explain the neurologic deficits that 522
9. Where will the motor and sensory deficit manifest (below the head) if there is a lesion of the internal capsule? 523
Case17-1 524
1. How might we approach a case of suspected motor neuron disease? 524
Case17-1 continued: 524
2. What upper motor neuron signs are present in this patient? 524
3. Why are the signs of hyperreflexia, spastic paralysis, and clonus seen with an upper motor neuron lesion? 525
4. What lower motor neuron signs are present in this patient? 525
Case17-1 continued: 525
5. What process leads to the findings seen in this biopsy specimen stained with hematoxylin-eosin (H&E)? 526
6. What would myosin adenosine triphosphatase (ATPase) staining of this specimen show? 526
7. What is the most likely diagnosis? 526
8. Why are the magnetic resonance imaging and cerebrospinal fluid findings notable? 526
9. What are the principal pathologic findings in amyotrophic lateral sclerosis? 526
10. What might you expect electromyography and nerve conduction studies to show in this patient? 527
11. Is amyotrophic lateral sclerosis more commonly inherited or acquired? 527
12. Why is amyotrophic lateral sclerosis often confused for syringomyelia and vice versa? 527
Case17-2 528
1. With what actions is this tremor most likely to appear? 528
2. Differentiate among resting tremor, intention tremor, and postural tremor. 528
Case17-2 continued: 528
3. In light of these signs, what is the most likely diagnosis for the tremor? 528
4. How does a festinating gait differ from an ataxic gait? 529
5. Why should we determine whether this patient is taking medications such as haloperidol or metoclopramide? 529
6. What cerebral structures are affected in Parkinson's disease, and how does this play into the bradykinesia and akinesia ob 529
Case17-2 continued: 530
7. What medication did you start the patient on, and why is it, rather than dopamine, used to treat Parkinson's disease? 530
8. Why is levodopa typically administered along with carbidopa? 530
9. Drugs such as bromocriptine and pergolide are also used to treat Parkinson's disease. How do they exert their effects? 530
10. What is the mechanism of action of selegiline, a drug used in treating Parkinson's disease? 530
11. Why is it preferable to selectively inhibit monoamine oxidase B, rather than both monoamine oxidase A and monoamine oxida 530
12. What is benztropine and why is it useful in Parkinson's disease? 530
13. Which antiviral medication is also effective in treating Parkinson's disease? 530
14. How does the drug MPTP (1-methyl-4-phenyl-1,2,3,6-tetrahydropyridine) cause parkinsonism, and is this a reversible proces 531
15. Why should you be suspicious of a diagnosis of Parkinson's disease in a patient being treated for schizophrenia? 531
16. What would a pathologist look for to establish the diagnosis of Parkinson's disease in evaluation of the brain at autopsy 531
Case17-3 531
1. What term describes the patient's movements, and what conditions may cause these movements? 531
2. Why might medications such as haloperidol and l-dopa cause chorea? 532
Chapter 18: Ophthalmology 568
Basic concepts 568
1. Describe the course of visual information arriving from the left and right visual fields. 568
2. What visual field defect results from midline sectioning of the optic chiasm? Explain. 569
3. What visual field deficit will occur with sectioning of the left optic tract and why? 570
4. What visual field deficit is likely with a tumor in the right temporal lobe? 570
5. A physician shines a light into a patient's right eye and notes bilateral constriction of both pupils (normal response). D 571
6. If there is a lesion in the left optic nerve, what would be the pupillary response if a light is shone into the right eye 571
7. What will the pupillary response be to shining a light in either eye if there is a lesion in the left oculomotor nerve? 572
8. What is an Argyll Robertson pupil? 572
9. If the oculomotor nerve is paralyzed on one side, why is the eyeball on that side rotated laterally and inferiorly (``dow 572
10. Describe the function of the medial longitudinal fasciculus. 573
11. What is pathologic nystagmus? 573
Case18-1 574
1. What is the most likely diagnosis? 574
2. What is the difference between open-angle glaucoma and closed-angle glaucoma? 574
3. What is the mechanism by which beta blockers reduce intraocular pressure in glaucoma? 575
4. What is the mechanism of action by which topical and oral carbonic anhydrase agents (e.g., acetazolamide) could be used to 575
Related Questions 575
5. Why are cholinomimetics like pilocarpine and carbachol useful for closed-angle glaucoma? 575
6. Why should epinephrine be avoided in closed-angle glaucoma? 575
Case18-2 576
1. What is the most likely diagnosis? 576
2. What is the difference between ``dry´´ and ``wet´´ age-related macular degeneration? 576
3. Where is the macula and what is its function? 576
4. What is the mechanism of action for the intravitreal injection of ranibizumab that will be used to treat this man's age-re 576
5. What simple diagnostic test can be carried out to detect age-related macular degeneration? 576
Chapter 19: Rheumatology 577
Basic concepts 577
1. What is a ``diarthrodial´´ joint? 577
2. What are the components of a diarthrodial joint and which sites within a diarthrodial joint are vulnerable to disease? 577
3. What are synarthrodial and amphiarthrodial joints? 577
Case19-1 579
1. What is the differential diagnosis? 579
Case19-1 continued: 579
2. What is the likely diagnosis? 579
3. Is the pathogenesis of this condition primarily related to degeneration of bone, cartilage, or synovial membrane? 580
4. What is the anatomic source of the joint pain in osteoarthritis? 580
5. What are some risk factors associated with developing osteoarthritis? 580
6. Would you expect the erythrocyte sedimentation rate (ESR) or C-reactive protein (CRP) to be elevated in this patient? 581
7. How do the findings on x-ray studies generally differ between osteoarthritis and rheumatoid arthritis? 581
8. Nonsteroidal anti-inflammatory drugs and cyclooxygenase-2 inhibitors are both possible treatments for this patient. What i 582
9. What is the principal therapeutic advantage of the cyclooxygenase-2 inhibitors? What can be given to this patient with a n 582
10. Why are cyclooxygenase-2 inhibitors considered dangerous in some patients? 582
11. Is acetaminophen a reasonable option for treating joint pain in this patient? 582
12. What sort of analgesic would you prescribe for a patient with preexisting renal disease? Or liver disease? Or pregnancy? 582
13. Quick review: Look only at the left column in Table19-1 and try to list the class of drug, mechanism of action, and majo 583
Case19-2 583
1. What is the differential diagnosis? 584
Case19-2 continued: 584
2. What is the diagnosis? 584
3. What are the criteria for the diagnosis of rheumatoid arthritis? 584
4. What sort of damage occurs in the joints of patients with rheumatoid arthritis? 585
5. What is the significance of rheumatoid factor in this disease? 585
6. What is the epidemiology of rheumatoid arthritis? 585
7. What is the typical treatment for rheumatoid arthritis? 585
8. What would be learned from aspiration of this patient's knee? 585
Case19-2 continued: 586
9. What are the characteristic deformities in the hands in advanced rheumatoid arthritis? 586
10. What are the extra-articular manifestations of rheumatoid arthritis? 587
Case19-3 587
1. What is your differential diagnosis? 587
2. How can crystal and septic arthritis be differentiated? 587
Case19-3 continued: 588
3. What is the likely diagnosis? 588
4. What is the pathophysiology of this condition? 588
5. Do most people with hyperuricemia develop gout? 588
6. What is Lesch-Nyhan syndrome and why might it predispose to gout? 588
7. Are there any complications of gout besides the monoarticular inflammatory arthritis? 589
8. Are there any medications that can precipitate attacks of gout? 589
Case19-3 continued: 589
9. What other treatment options exist for an acute gout attack? 589
10. Who needs chronic treatment for gout, and what does this treatment consist of? 590
11. What is pseudogout? 590
One Last Crystal 590
12. What is going on here? 590
Case19-4 591
1. What is your differential diagnosis for diffuse musculoskeletal pain and fatigue? 591
Case19-4 continued: 591
2. What is the likely diagnosis? 592
3. What causes fibromyalgia? 592
4. Are steroids indicated? 592
5. Who is prone to developing fibromyalgia? 592
Case19-5 593
1. What is the diagnosis in this patient? 593
2. How is this condition treated? 593
3. What else should be looked for in patients with suspected polymyalgia rheumatica? 593
Case19-6 594
1. What is your differential diagnosis? 594
Case19-6 continued: 594
2. What is the Phalen sign? What is the Tinel sign? 594
3. What is your diagnosis? 594
4. What causes carpal tunnel syndrome? 594
5. What structures pass through the carpal tunnel? 595
6. What does the median nerve innervate in the hand? 595
7. What are treatment options for carpal tunnel syndrome? 595
Case19-7.1 596
1. What is the diagnosis? 596
2. What is the bamboo spine? 597
Case19-7.2 597
1. What is the likely diagnosis? 597
2. What triggers this condition? 597
Case19-7.3 597
1. What is the diagnosis here? 597
2. What joints are affected by psoriatic arthritis? 597
Case19-8 599
1. What do you think is going on here? 599
Case19-8 continued: 599
2. What is your diagnosis? 599
3. What is the pathogenesis of scleroderma? 600
4. Which sort of scleroderma does this patient have? 600
5. What is the reason for the reflux? 600
CASE19-8 continued: 600
6. What are the renal manifestations of diffuse scleroderma? 600
7. What is the current leading cause of death for these patients? 601
8. What is the CREST syndrome? 601
9. What is localized scleroderma? 601
10. How is scleroderma treated? 601
11. Quick review: Cover columns 2, 3, and 4 in Table19-2 and describe the various characteristics of diffuse, limited, and l 602
Case19-9 603
1. What is the likely diagnosis in this patient? 603
2. How can a diagnosis of discoid lupus be confirmed? 603
3. What treatment options exist for discoid lupus? 603
Case19-9 continued: 603
4. What is the likely diagnosis? 604
5. What is the pathogenesis of systemic lupus erythematosus? 604
6. What causes lupus? 604
7. What are the criteria for diagnosing systemic lupus erythematosus? 604
8. What are ``antinuclear antibodies,´´ and are they sensitive or specific for systemic lupus erythematosus? 604
9. What is causing this patient's dry mouth and dry eyes? 604
10. What is the reason for the proteinuria? 605
11. What is the reason for this woman's anemia? 605
12. Why did this patient have a positive Venereal Disease Research Laboratory (VDRL) test? 605
13. What are the treatment options for systemic lupus erythematosus? 605
14. What medications are responsible for drug-induced lupus? 605
15. Quick review: Cover the right column in Table19-3 and give the primary disease(s) associated with the autoantibodies lis 606
Case19-10 607
1. What is your differential diagnosis? 607
Case19-10 continued: 607
2. What is the diagnosis? 608
3. What causes this disease? 608
4. What is the treatment? 608
5. What are the other ``idiopathic inflammatory myopathies´´? 608
6. What disease that is transmitted by pork can cause similar muscular symptoms? 608
Case19-11 609
1. What is your differential diagnosis? 609
Case19-11 continued: 609
2. What is the most likely diagnosis? 609
3. Is this condition more commonly acquired or inherited? 609
4. What is the function of dystrophin? 610
5. How do the manifestations of Becker's muscular dystrophy differ? 610
6. Why does this boy have such large calf muscles on examination? What term is used to describe this finding in Duchenne's mu 610
7. How is a Gowers´ sign elicited on examination and what does it indicate? 611
A Few More Muscular Dystrophies 611
8. What diagnosis might you suspect? 611
9. What is the mechanism of inheritance of myotonic dystrophy? 611
10. What is the diagnosis? 612
Case19-12 612
Case19-12.1 612
1. What is your diagnosis? 612
2. What causes the bones to be dense and thick in this patient? 613
3. Why might you see anemia in osteopetrosis and why is it referred to as a myelophthisic anemia? 613
4. Should you observe any laboratory value abnormalities in a patient who has osteopetrosis? 613
Case19-12.2 614
5. Infection with what organisms should be suspected? 614
Case19-12.3 614
6. What most likely explains this? 614
Case19-12.4 614
7. What is your diagnosis and what is the etiology of this condition? 614
8. Quick review: Cover the right column in Table19-4 and list the pathophysiologic abnormality associated with each of the 615
Chapter 20: Vasculitides 618
Basic Concepts 618
1. What are the vasculitides and how do they typically present clinically? 618
2. Along with constitutional complaints, what clinical signs and patterns of organ involvement suggest a vasculitic syndrome? 618
3. How are the vasculitides classified? 619
4. Cover the right column in Table20-1 and attempt to describe the ``classic presentation´´ for each of the liste 619
Case20-1 620
1. What are the main considerations in your differential diagnosis? 620
Case20-1 continued: 620
2. What is the likely diagnosis? 620
3. What does the elevated erythrocyte sedimentation rate imply? 620
4. In temporal arteritis, what events lead to inflammation of the artery? 621
Case20-1 continued: 621
5. Why might it still make sense to treat this patient? 621
6. What severe complication of this disorder may be avoided by initiating immunosuppressive therapy as soon as possible? 621
7. What other symptomatic manifestations may be expected as a result of arterial inflammation in patients with giant cell art 621
8. How can response to corticosteroids be monitored? 621
9. What other disease is giant cell arteritis associated with? 622
Case20-2 622
1. What are the considerations in your differential diagnosis? 622
Case20-2 continued: 623
2. What is your diagnosis? 623
3. What is the major concern in patients with this disease who do not receive adequate therapy? 623
4. What is the pathogenesis of Kawasaki disease? 624
5. How is this disease treated? 624
6. Describe the epidemiology of this disease. 624
Case20-3 625
1. What is your differential diagnosis? 625
Case20-3 continued: 625
2. What is the likely diagnosis? 625
3. What causes disease manifestations? 625
4. What is the usual progression of symptoms in this disease? What other organ systems will likely become involved? 625
5. How is Wegener's granulomatosis treated? 626
Case20-4 626
1. What is the diagnosis? 626
2. What is the significance of the positive hepatitis B surface antigen in this patient? 627
Case 20-3 continued: 627
3. What are the dermatologic manifestations of polyarteritis nodosa? 628
4. How can the diagnosis of polyarteritis nodosa be confirmed? 628
Case20-4 continued: 628
5. Given the high-dose steroids, what prophylaxis needs to be considered? 628
Case20-5 629
1. What is the likely diagnosis in this patient? 629
2. What is Takayasu's arteritis? 629
3. What makes this patient different from the typical presentation of Takayasu's arteritis? 629
4. Ischemic complication due to vascular involvement of the arch of the aorta and its major branches led to this patient's sy 630
5. How is the diagnosis made? 630
6. How is this disease treated? 630
Chapter 21: Bacterial Diseases 631
Basic Concepts 631
1. What makes an organism gram-positive or gram-negative? 631
2. Why are gram-negative infections more likely to produce bacterial sepsis? 631
3. Describe the mechanism by which lipid A causes toxicity? 632
4. What are exotoxins? 632
5. What is a capsule and what purpose does it serve? 633
6. What sort of individuals are susceptible to infection by encapsulated bacteria? 633
7. Identify the Gram stain and the morphology of the organisms in Table 21-1. 634
8. Cover the two columns on the right in Tables 21-2 through 21-9 to test your knowledge of the properties of the clinically 635
Basic Concepts in Antibacterial Pharmacology 637
1. What are the β-lactam antibiotics and what is their mechanism of action? 637
2. Why are clavulanic acid and sulbactam added to some penicillins? 639
3. What is the antibacterial spectrum of the various subclasses of penicillins and cephalosporins (Table 21-10)? 639
4. What is the antibacterial spectrum of the fluoroquinolones and what is their mechanism of action? 641
5. What is the spectrum and mechanism of action of the macrolides? 643
6. What is special about the tetracyclines? 643
7. What are the mechanism of action and spectrum of the aminoglycosides? 643
8. How does chloramphenicol work and why is it not used more often? 643
9. Why is trimethoprim commonly given in combination with sulfamethoxazole, as TMP-SMX? 643
10. Cover the two columns on the right of Table 21-11 and describe the mechanism of action and mechanism of bacterial resista 643
11. Cover the right column in Table 21-12 and describe the adverse effects for each of the antimicrobial agents listed in the 644
Case21-1 645
1. What is the most likely diagnosis? 646
2. What defense mechanisms prevent pneumonia in the healthy individual? 646
3. Why might a patient in the intensive care unit who is intubated be at increased risk for developing pneumonia? 646
4. Why is it important to distinguish between community-acquired and nosocomial pneumonia? 646
5. What is atypical (``walking´´) pneumonia, and is the patient in this case more likely have a typical or an atypical 646
Case21-1 continued: 647
6. What is the diagnosis? 648
7. How should this patient be treated pharmacologically? 648
8. Use Table 21-13 to quiz yourself on the most common causes of pneumonia in different age groups. 648
9. Use Table 21-14 to quiz yourself on the important characteristics of the organisms that are known to cause pneumonias. 648
Case21-2 650
1. What is the most likely diagnosis? 650
2. What other types of diarrhea can be caused by Escherichia coli? 650
3. What is the difference between osmotic and secretory diarrhea? Name a cause for each type. 651
4. What predisposes to Clostridium difficile colitis and what sort of diarrhea does this cause? 652
5. How is diarrhea treated? 652
Case21-3 654
1. What is the most likely diagnosis? 654
2. What are the major risk factors for developing endocarditis? 656
3. What are the clinical signs of bacterial endocarditis? 656
4. What are the clinical signs of rheumatic fever? 656
5. Which bacteria are most commonly associated with bacterial endocarditis? 658
6. What drugs could be used to treat this patient? 658
7. How does bacterial endocarditis differ from Libman-Sacks endocarditis? 658
Case21-4 659
1. What is the likely diagnosis? 659
2. Based on this man's presentation, in which ``stage´´ of syphilitic infection is he most likely to be? 659
3. What stage of syphilis would you suspect in a patient with a diffuse maculopapular rash? 659
Case21-4 continued: 660
4. What is the likely diagnosis? 660
5. Use Table 21-21 to quiz yourself on the three stages of syphilitic infection. 660
6. What diagnostic tests could be done to definitively diagnose syphilis in this man? 660
7. How would you treat this patient? 660
8. Later that night, the patient calls you at home with serious concerns about a reaction to penicillin. He states that sever 662
Case21-5 662
1. What is the most likely diagnosis? 663
2. What stage of Lyme disease would you suspect in this child? 663
3. How would your diagnosis change if this patient presented with a similar history but had complaints of various painful swo 663
4. If this patient does not receive appropriate treatment, what is the likelihood that the infection will progress to stage 3 664
5. What is the treatment for Lyme disease? Name a preventive measure that can be taken. 664
6. Describe the Ixodes life cycle. 664
Case21-6 665
1. What is the presumptive diagnosis? 665
2. How is this disease primarily transmitted? 665
3. Why is the acid-fast stain required to visualize this bacterium? 666
4. Does this patient most likely have primary tuberculosis, latent tuberculosis, or recrudescent (secondary) tuberculosis? 666
5. What are the first-line drugs for treating tuberculosis and why are they always used in combination? 667
6. If this patient is treated with isoniazid as part of his regimen, why should he also receive supplemental pyridoxine (vita 667
7. If this patient is treated with rifampin as part of his regimen, why may he need larger doses of opioid analgesics for pai 667
8. Three weeks after starting a therapeutic regimen with rifampin and isoniazid the patient complains of orange urine. What i 667
9. If this patient begins complaining of vision problems, what would you suspect is the cause? 667
10. Why is the standard treatment regimen that this patient will be put on so prolonged? 667
Related Questions 667
11. Is cell-mediated immunity or humoral immunity more important for fighting tuberculosis? Why? 667
12. How does the purified protein derivative skin (Mantoux) test work? 667
13. Why is reactivation tuberculosis more likely to occur in the apical lungs rather than in the lower lobes? 668
14. What type of necrosis is associated with granulomatous cell death in tuberculosis? 668
15. What type of secondary infection can be seen in pulmonary cavitation such as that associated with tuberculosis? 668
16. How can tuberculosis cause a urinalysis to show microscopic pyuria and hematuria (with red blood cell casts) in the face 668
17. Why might Pott's disease be suspected in a patient with tuberculosis who has new-onset back pain but denies any trauma th 668
Case21-7 668
1. What is the most likely diagnosis? 669
2. What is unique about the chlamydial cell wall? 669
3. When a sample of infected tissue is stained with Giemsa, where will the chlamydial bacteria be seen? 669
4. How is pelvic inflammatory disease transmitted and why can it lead to pelvic discomfort, vaginal discharge, and vaginal bl 669
5. What are the two organisms that could most likely cause her symptoms? 669
6. What test would you do to differentiate between these two organisms and why? 669
7. How are Chlamydia trachomatis and Neisseria gonorrhoeae transmitted? 669
Case21-7 continued: 669
8. What should be prescribed as a treatment for your patient? 669
9. If the patient's current and past partners do not have any symptoms, should they also be considered for treatment? 669
10. Why is the fact that the patient was using an intrauterine device significant in this case? 670
11. What are other risk factors for the development of pelvic inflammatory disease? 670
12. If this patient was not using any birth control and had been trying to become pregnant, what other concerns would you nee 670
13. What is Reiter's syndrome? 670
14. Describe the unique life cycle of a chlamydial infection. 670
15. What are the serotypes of Chlamydia trachomatis that can cause pelvic inflammatory disease? 671
16. What are the other species of Chlamydia and what diseases do they cause? 671
Case21-8 671
1. What is the most likely diagnosis? 672
2. What is the ``classic triad´´ of symptoms associated with meningitis? 672
3. What are the Brudzinski and Kernig signs? 672
4. What are the possible causes for meningitis? What tests can be done to make the diagnosis of meningitis and identify the c 672
5. What are the most common causes of meningitis by age group? Use Table 21-27 to quiz yourself. 672
6. Haemophilus influenzae used to be the most common cause of meningitis in newborns but is now only rarely seen in this age 672
7. When would be an appropriate time to initiate antibiotic therapy in this patient and what antimicrobial agent could be use 673
8. A Gram stain of the cerebrospinal fluid shows gram-negative cocci in pairs. What is the most likely cause of the meningiti 673
9. In a patient with human immunodeficiency virus (HIV), what infective agents may be more likely to cause meningitis than in 673
Chapter 22: Viral, Parasitic, and Fungal Diseases 674
Basic concepts in virology 674
1. What structural components are used to categorize viruses? 674
2. Name the disease associated with each DNA virus listed in Table 22-1. 676
3. Cover the right-hand column in Table 22-2, and using the clinical description given, name the most likely virus. 677
Basic concepts in parasitology 678
1. What are protozoa? 678
2. What is the difference between cestodes, nematodes, and trematodes? 679
3. Cover the left column in Table 22-4, and from the description of the infection at the right, name the helminth that causes 679
Basic concepts in mycology 680
1. What are the two morphologic types of pathogenic fungi? 680
2. What is meant by the term dimorphic fungi? 680
3. How do the antifungal ``-azole´´ agents work? 681
4. What is the mechanism of action for amphotericin B and nystatin? 681
5. Cover the right-hand column in Table 22-5 and determine the most likely fungal organism based on the clinical description 681
Case22-1 682
1. With this initial history, what is your differential diagnosis? 682
Chapter 23: Pharmacology and Toxicology 693
Basic Concepts 693
1. How does the route by which a drug is administered affect its metabolism? 693
2. What does the Henderson-Hasselbalch equation mean? 694
3. What is the difference between zero-order kinetics and first-order kinetics? 694
4. What is half-life? 695
5. What is the difference between loading dose and maintenance dose and how is this affected by liver and renal disease? 695
6. What is therapeutic index? 695
7. What are the differences between competitive and noncompetitive inhibitors? 695
Case23-1 696
1. Given the preceding clinical picture, what is the most likely explanation for this patient's presentation? 696
2. What are the pharmacotherapeutic actions of aspirin and other nonsteroidal anti-inflammatory drugs? 696
3. What is the mechanism of action of NSAIDs? 697
4. What are some side effects of NSAIDs, and what alternative medications exist that circumvent these side effects? 697
5. If given the choice of aspirin or acetaminophen, which would you administer to a child with a fever. 697
Case23-2 698
1. On the basis of the preceding information, what can be the cause of this patient's presentation? 698
2. What is the mechanism of action of opioids? 698
3. List a few members of the opioid family. 699
4. What are some signs/symptoms of opioid intoxication? Is it life-threatening? 699
5. How can an opioid overdose be reversed? 699
6. What is the difference between naloxone and naltrexone? 699
7. What role can methadone play in treating opioid dependence? 699
8. What are signs/symptoms of opioid withdrawal? 699
Case23-3 700
1. Given this clinical picture, what is the most likely explanation for this patient's presentation? 700
2. What is the mechanism of action of cocaine? 700
3. The net effects of cocaine on the body can mimic those of which other illicit drug? 700
4. What is the explanation for development of tolerance to cocaine use? 700
5. Aside from tolerance and dependence, what are some other adverse effects of cocaine? 700
Amphetamines 701
6. How does the mechanism of action of amphetamines differ from that of cocaine? 701
7. What are some adverse effects of amphetamines? 701
Lysergic Acid Diethylamide (LSD) 701
8. What is the mechanism of action of LSD and its effects on the body? 701
Phencyclidine (PCP) 701
9. What is the mechanism of action of phencyclidine and its resultant effects on the body? 701
Tetrahydrocannabinol (THC) 701
10. What is the mechanism of action of tetrahydrocannabinol and its resultant effects on the body? 701
Case23-4 702
1. Based on this presentation, what is the likely culprit? 702
2. What is the mechanism of action of digitalis? 702
3. What are clinical indications for using glycosides? 703
4. What are some adverse effects of glycosides? 703
Case23-5 704
1. In general terms, how does a caustic agent damage tissue? 704
2. How do alkaline agents damage cells, tissues, and organs? 704
3. How do acidic agents damage cells, tissues, and organs? 704
4. If you encountered the preceding patient in the emergency department, what are the next few steps that would need to be ad 704
Case23-6 705
1. What is likely causing this patient's symptoms? 705
2. What is the mechanism of action of calcium channel blockers? 705
3. What are the different types of calcium channel blockers? 705
4. What are some general side effects of calcium channel blockers? 707
Case23-7 707
1. Given the preceding presentation, which types of items are at the top of the differential diagnosis and would be worth exp 707
2. What is the mechanism of action of benzodiazepines? 708
3. What are a few clinical indications for using benzodiazepines? 708
4. What are some common adverse effects of benzodiazepines? 708
5. What are symptoms of benzodiazepine intoxication and withdrawal? 709
Case23-8 709
1. Given his recent uncomplicated hospital course and his benign past medical history, what condition is this patient likely 709
2. What are the symptoms of acute alcohol toxicity and how can these effects be explained at the molecular level? 710
3. What are symptoms of chronic alcohol abuse? 710
4. What is the relationship between alcohol and benzodiazepines in terms of their effect on the brain? 710
5. How is alcohol metabolized in the body? 710
Case23-9 711
1. Given this presentation, which other laboratory values would prove informative? Should this patient be treated based on th 711
2. Is acetaminophen considered a nonsteroidal anti-inflammatory drug? 711
3. What is the mechanism of action of acetaminophen? 712
Case23-10 712
1. Given this patient's presentation, which group of medications is the likely culprit? 712
2. In general terms, how is the nervous system organized? 713
3. Which neurotransmitter can be said to be pivotal to the function of the entire motor nervous system? 713
4. Is there a way to selectively affect the parasympathetic nervous system? 714
5. What is one way to reduce the side effects of a drug that stimulates both nicotinic and muscarinic receptors? 714
6. Are there nicotinic receptor blockers that are selective for the autonomic nervous system or somatic nervous system rather 714
7. How can a cholinergic drug help diagnose myasthenia gravis? 715
Case23-11 716
1. Given this clinical picture, what scenario best explains this patient's presentation? 716
2. What is the mechanism of action of barbiturates? 716
3. What are some common indications for using barbiturates? 716
4. What are some common adverse effects of barbiturates? 717
Case23-12 717
1. Given this presentation, to which group of antihypertensives does the likely culprit belong? 718
2. In the sympathetic nervous system, what are the two types of neurotransmitters and the two main adrenergic receptors? 718
3. Describe α-receptors in terms of their distribution in the body and a few of their agonists/antagonists. 718
4. Describe β-receptors in terms of their distribution in the body and a few of their agonists/antagonists. 719
Case23-13 720
1. What is the likely cause of this man's symptoms? 720
2. How does organophosphate poisoning result in this patient's symptoms? 720
3. What is the treatment for organophosphate poisoning? 721
4. Atropine administration alone relieves which of the DUMBBELSS symptoms? 721
Chapter 24: Behavioral sciences 722
Case24-1 722
1. What are the considerations in the differential diagnosis? 722
2. In addition to a diagnosis of adjustment disorder with depressed mood, what sleep-related disorder likely explains most of 723
3. What treatment can be employed to allow this man to sleep at night? 723
Case24-2 723
1. What are the considerations in the differential diagnosis? 723
2. What is the likely diagnosis and what would be the expected electroencephalographic findings? 724
3. Which treatment is available for patients with narcolepsy? 724
4. What are the stages of sleep and what happens physiologically in these stages? 724
5. In Table24-1, cover the columns to the right, and for each stage of sleep listed in the left column, name the EEG appear 724
6. How do nightmares differ from night terrors? 725
7. An 82-year-old woman complains that her sleep patterns have changed as she has aged. What changes in sleep are typical as 725
8. This woman had been given a benzodiazepine to assist her sleep, which improved for a while, but now she complains of poor 725
9. How do benzodiazepines manifest their pharmacologic effect? 725
10. Why would this be another reason benzodiazepines should be avoided in the elderly population? 725
11. There are now a number of drugs other than benzodiazopines that also act on the γ-aminobutyric acid benzodiazepine recep 726
12. When evaluating a person for sleep problems, perhaps the first and most important step is to make sure that the patient h 726
Case24-3 726
1. What should the physician do? 726
2. What are advance directives? 726
3. How is competence (decision-making capacity) defined? 727
4. What is substituted judgment? 727
Case24-4 727
1. What should you do? 727
2. What are the rules regarding parental consent for minors? 727
Case24-5 728
1. What do you do? 728
2. Is it a good idea for you to be honest and tell her that you cannot have a relationship with her while she is your patient 728
Case24-6 729
1. What do you do? 729
2. Under what other conditions is it acceptable to violate patient confidentiality? 729
Case24-7 729
1. What is the differential diagnosis for this patient's sexual dysfunction? 729
2. What changes occur in the elderly with regard to sexual health? 730
Case24-8 730
1. What should you do? 730
Case24-9 730
1. What do you do? 731
2. What is the CAGE questionnaire? 731
Case24-10 731
1. Is this child developing normally? 731
2. What should you tell this concerned parent? 731
Case24-11 733
1. How should you handle this situation? 733
2. How can compliance be increased in the future? 733
Case24-12 734
1. How should you handle this situation with the patient's wife? 734
2. What should you say to the patient when you walk into the room? 734
Case24-13 734
1. What is the most likely diagnosis? 734
2. What are the most common symptoms of restless legs syndrome? 735
3. How is restless legs syndrome treated? 735
Chapter 25: Biostatistics 736
Basic concepts 736
Test Characteristics 736
1. What does the sensitivity of a diagnostic test measure? 736
2. What does the specificity of a diagnostic test measure? 737
3. Quick terminology review: Cover the right column in Table25-2 and define each of the terms in the left column. 737
4. How does the sensitivity of a test relate to its specificity? 737
5. What information is given by the relative risk? 738
6. What information is given by the odds ratio? 738
7. What is the difference between probability and odds and how are they measured? 738
8. What is the positive predictive value? negative predictive value? 739
9. What is the positive likelihood ratio? 739
10. How is the positive likelihood ratio used to calculate the positive predictive value? 739
11. What is meant by the reliability of a test? 740
12. What is meant by the validity of a test? 740
13. In statistical analyses of differences between groups, a P value is often included to reflect how significant the differe 740
14. What are the differences between type I and type II error? How is power related to type II error? 740
15. What are some determinants that can be used to evaluate the existence of a causal relationship between two variables? 741
16. What is the difference between prevalence and incidence? 741
17. How do the incidence and duration of a disease affect its prevalence? 741
Measures of Spread 741
18. The following sample distribution pattern lists the ages of 11 patients seen by a physician on a given day: 741
19. What does the standard deviation of a population represent? 743
Study Designs 744
20. What is meant by the term ``bias´´ and which study design best eliminates bias? 744
Case25-1 744
1. What sort of study design was this? 744
2. How does a retrospective case-control study differ in design from a retrospective cohort study? 744
3. What are the strengths of a case-control study? 746
4. What are the limitations of a case-control study? 746
5. What ratio can be used to compare event rates in a case-control study? 748
6. How is the odds ratio calculated? 748
Case25-2 749
1. What does the expression ``13010mm Hg´´ mean with respect to the distribution of blood pressure in this sam 749
2. What does it mean when the blood pressure in this population is said to be ``normally distributed´´? 749
3. What does a small coefficient of variation for the sample in the blood pressure study imply? 750
4. Quick review: What percentage of the men in this study had a blood pressure in the following ranges? 750
case25-3 751
1. Why does it make sense to use a screening test with a high sensitivity, even at the cost of specificity, for this patient? 751
2. If this patient tests positive, is it reasonable to tell him that you are 95% confident that he is infected with human 751
3. What if the test comes back negative? Can you tell this patient that you are 75% confident that he does not have human 751
4. Now let's assume that 90-year-old grandma and our young drug-abusing model citizen in this vignette both test positive for 751
Case25-4 752
1. Given the data presented in the 22 table in Table 25-7, what is the sensitivity of this new test? 752
2. What is the specificity of this new test? 752
3. What information can be obtained from calculating the positive likelihood ratio? 752
4. How is the positive likelihood ratio used to calculate the positive predictive value? 753
5. If the positive likelihood ratio is not known, what is another way to calculate the positive predictive value? 753
6. How is the negative predictive value calculated? 753
7. Using the same preceding example for the calculation of the positive predictive value, calculate the negative predictive v 753
Case25-5 754
1. What type of study design is this? 754
2. What is the difference between a prospective cohort study and a retrospective cohort study? 754
3. What is the major limitation of cohort studies? 754
4. On the basis of data presented in Table 25-8, what is the relative risk for cancer in the exposed group? 755
5. What is meant by attributable risk and attributable risk percent? Calculate both for the preceding example. 755
6. What experimental design overcomes the shortcomings of the cohort study? 756
Chapter 26: Clinical Anatomy 757
Case26-1 757
1. What is the differential diagnosis for his foot and lower extremity pain? 758
Case26-1 continued: 758
2. What is now the most likely diagnosis? 758
3. Which historical features in this patient increase the likelihood of a peripheral vascular disease diagnosis? 758
4. Why does his nocturnal right foot pain resolve when he hangs the affected foot over the bedside? 758
5. Describe the path of arterial blood from the heart to the femoral sheath. 758
6. Outline the borders of the femoral triangle. 758
7. Describe the path of arterial blood from the femoral sheath to the feet. 759
8. At which sites is arterial plaque formation most likely? 759
9. In Table26-1, cover the two columns on the right and attempt to list the drug class and mechanism of action for each of 760
Case26-2, Part A 761
1. What structure has Oscar injured, and how has it led to his upper extremity position? 762
2. If Oscar had forced his upper extremity above his head by grabbing the handlebars of the motorcycle to prevent his fall, h 762
Case26-2, Part A continued: 762
3. What structure has Oscar injured this time and how has it led to his upper extremity position? 763
Case26-2, Part B 763
4. What structure has been injured and how has this happened? 763
5. Outline the contents of the carpal tunnel. 763
6. Name the muscles of the thenar and hypothenar eminences and describe their function. 763
7. Describe the pattern of sensory innervation of the hand. 763
Case26-2, Part B continued: 764
8. What fracture has George suffered? 764
Case26-2, Part C 764
9. What are the three most common sites of humerus fracture, and which nerve and artery are at risk at each of these sites? 764
10. On reviewing Jake's past medical history, you note that his baseball career was marred by a partially torn rotator cuff. 766
Case26-3, Part A 767
1. What is the most likely diagnosis? 767
2. What is the utility of the ductus arteriosus? 767
3. What causes the ductus arteriosus to close after birth? 767
Case26-3, Part B 769
4. What is the most likely diagnosis? 769
5. Is atrial septal defect the most common congenital heart defect? 769
6. What are the three most common types of atrial septal defect? 769
7. How might atrial septal defect lead to right-sided heart failure? 769
8. What is the dreaded late complication of atrial septal defect? 769
Case26-3, Part C 770
9. What are the five cardiogenic causes of cyanosis in a newborn? 770
Case26-3, Part D 770
10. What is the most likely diagnosis? 770
11. How does tetralogy of Fallot cause cyanosis? 771
12. What is the cause of tetralogy of Fallot? 771
Case26-4, Part A 772
1. What is the differential diagnosis for John's symptoms? 772
Case26-4, Part A continued: 772
2. What is the most likely diagnosis and what is the next step to confirm this suspicion? 772
3. At what spinal level should a lumbar puncture be performed? Why? 772
4. Through what major structures and spaces, from superficial to deep, should the needle pass in a lumbar puncture? 772
5. Describe the three layers of the meninges. 773
6. What cerebrospinal fluid findings would you expect to find with different causes of meningitis? 774
7. What are the most common causes of meningitis by age group? 774
Case26-4, Part B 774
8. What disorder of neurologic development can be characterized by these findings? 775
9. What are the other significant disorders related to a failure of posterior neural tube closure? 775
Case26-4, Part C 775
10. What is the most likely diagnosis? 775
11. Describe intervertebral disk anatomy and how herniation usually occurs. 775
12. Describe the pattern of nerve compression seen in intervertebral disk herniations. 775
Case26-5 777
1. Describe the characteristics of the most likely diagnosis. 778
2. What differentiates a direct from an indirect inguinal hernia? 778
3. Describe the structure of the inguinal canal. 778
4. Discuss the major contents of the spermatic cord. 779
5. Which lymph nodes are the most likely site of first metastasis in testicular cancer? Why is this the case? 779
6. After a vasectomy, by what means does a male produce an ejaculate that does not include sperm? Include a summary of the pa 780
7. Describe the neurologic basis for erection, emission, and ejaculation. 781
8. Name the most common drugs used for treatment of erectile dysfunction and outline their mechanism of action. 781
Case26-6 782
1. What is the differential diagnosis for Carlos's symptoms? 782
Case26-6 continued: 782
2. Which of the possibilities is now the most likely diagnosis? 782
3. Outline varicocele pathophysiology. Be sure to explain why varicocele is more likely to occur on the left than on the righ 782
4. Describe the difference between hydrocele and hematocele. 783
Case26-6 continued: 783
5. What are esophageal varices? 784
6. How do the esophageal veins connect the portal and systemic venous systems? 784
7. Outline the flow of blood to the superior vena cava through the veins of the azygous system. 784
Case26-7, Part A 785
1. List the structures that Nikolai has injured. 785
2. How does the posterior cruciate ligament differ from the anterior cruciate ligament? 786
3. Explain why Nikolai's medial collateral ligament and anterior cruciate ligament tears led to the tear in his lateral menis 786
Case26-7, Part A continued: 786
4. Nikolai's clinical picture suggests injury to what structure? 786
5. How does injury to the common fibular nerve result in footdrop, as seen in Nikolai? 786
6. List the muscles of the posterior compartment of the leg and describe their innervation. 788
Case26-7, Part A continued: 788
7. What dreaded vascular surgery complication has Nikolai suffered? 789
8. What divides the compartments of the leg? 789
9. Describe the major pathophysiologic characteristics of compartment syndrome. 789
Case26-7, Part B 789
10. What structure has been injured and how has this led to her awkward gait? 790
Case26-8, Part A 791
1. What is the differential diagnosis for Maureen's hoarseness? 791
2. Given the surgical history, which of these diagnoses is most likely and why? 791
3. How does injury to the recurrent laryngeal nerve result in hoarseness? 791
4. Describe the path of the recurrent laryngeal nerve, noting any asymmetries. 792
5. Describe the innervation of the lone intrinsic laryngeal muscle not innervated by the recurrent laryngeal nerve: the crico 792
Case26-8, Part B 792
6. To save this man's life, which structure must you incise? Why? 793
7. Describe the surface anatomy of the neck that allows one to find the cricothyroid membrane. 793
Case26-8, Part C 793
8. With what defect(s) has this child been born? 793
9. Describe the embryologic basis for cleft lip. 793
10. Describe the embryologic basis for cleft palate. 793
Other important concepts in embryology of the face and neck 793
11. Discuss the difference between pharyngeal (branchial) pouches, arches, and clefts. 793
12. In Table 26-5, cover the right column and name the derivatives of each structure listed in the left column. 795
13. From where do the parts of the thyroid gland, other than the parafollicular C cells, originate? 796
Case26-9 796
1. What is the differential diagnosis for Janice's right upper quadrant pain? 796
Case26-9 continued: 797
2. What is the most likely diagnosis? 797
3. Name the two most common organisms implicated in pelvic inflammatory disease. 797
4. How can pelvic inflammatory disease lead to right upper quadrant pain? 797
5. Describe how the uterus and ovaries are supported. 797
6. Can Fitz-Hugh-Curtis syndrome be seen in males? 797
7. Outline the common drugs used in antimicrobial pharmacotherapy for pelvic inflammatory disease. 797
8. In Table 26-6, cover the column on the right and name the abdominal organs in each location. 798
Case26-10, Part A 799
1. What is the differential diagnosis for David's chest pain? 800
2. A myocardial infarction in what distribution would be most concerning for damage to the sinoatrial and atrioventricular no 800
3. Which coronary arteries supply the left ventricle? 800
4. If David's chest pain were caused by pleuritis (also known as pleurisy), from which pleural layer would he be sensing pain 801
Case26-10, Part A continued: 801
5. What is the most likely diagnosis? 801
6. If one were to pass a needle from outside the pericardium to the lumen of the left ventricle, through which layers would i 801
7. Enlargement of which chamber of the heart is most likely to cause dysphagia? 801
8. Enlargement of which chamber of the heart is most likely to cause a parasternal lift? 801
Case26-10, Part B 801
9. If the candy passed into the bronchial tree, on which side would it most likely be found? 802
10. Describe the other major asymmetry of the bronchial tree. 803
11. List the four stages in lung development and note whether each is compatible with life. 803
Chapter 27: Pathology 805
1. A 67-year-old white man presented to the emergency department (ED) with worsening dyspnea and cough at night. Physical exa 805
2. A 62-year-old obese woman with a history of long-standing gastroesophageal reflux disease (GERD) presented to her primary 805
3. A 45-year-old homeless man, positive for human immunodeficiency virus (HIV), presented to the ED with reported chronic cou 806
4. A 35-year-old white woman presents to the ED with acute left lower quadrant abdominal pain that has been sharp and constan 806
5. A 60-year-old white man with a history of hyperlipidemia and diabetes mellitus type 2 presents to the ED with worsening ch 806
6. A 55-year-old white man presented to the ED after suffering severe injuries in a motor vehicle accident. His blood alcohol 806
7. A 46-year-old white man presents to his primary care physician with a 1-year history of fatigue, weakness, and arthralgias 807
8. A 74-year-old woman presents to her primary care physician complaining of constipation and hard stools for the past severa 807
9. A 60-year-old woman presents to her primary care physician complaining of feeling bloated and full for the past 2months no 808
10. A 38-year-old woman is referred by her primary care physician to a surgeon for treatment for a newly palpated, asymptomat 808
11. A 77-year-old white man presents to his primary care physician for his annual physical and was found to have a lesion on 808
12. A 71-year-old man presents to his primary care physician for the first time in over 10years with complaints of fatigue an 808
13. A 95-year-old man is brought to the ED for worsening SOB, fever, and productive cough. He is stabilized in the ED and adm 809
14. A 21-year-old woman presents to her gynecologist for her yearly Papanicolaou (Pap) smear. She has been sexually active wi 809
15. A 71-year-old man is brought to the ED with an acute exacerbation of congestive heart failure. He has had long-standing l 809
16. A 48-year-old woman is referred by her gynecologist to a surgeon for a newly palpated lump in her right breast that was n 809
17. A 15-year-old boy's father dies at the age of 50. An autopsy is performed, and the gross colon specimen is shown in Figur 809
18. A 62-year-old man develops severe chest pain that radiates down his left arm while eating dinner at home. He has a histor 810
19. A 63-year-old native Hawaiian woman presents to her primary care physician with worsening dull, aching back pain and fati 810
20. A 69-year-old white man presents to his primary care physician with a 4-week history of worsening cough productive of blo 810
21. A 26-year-old white man has a 2-week history of fevers that come and go, night sweats, and cervical and supraclavicular l 811
22. A 66-year-old man with a history of diabetes mellitus and many hospital admissions for nausea and vomiting was killed in 811
23. A 55-year-old African-American man with a history of GERD presents to his primary care physician with a 10-lb weight loss 811
24. A 72-year-old African-American man presents to his primary care physician with a 7-week history of worsening cough produc 812
25. A 55-year-old man presents to his primary care physician with a 3-week history of fever that comes and goes, a weight los 812
26. A 32-year-old man presents to his primary care physician complaining of a lump in his testicle that has been present for 812
27. A 65-year-old African-American man presents to his primary care physician with a 4-month history of fatigue and dyspnea o 813
28. An 85-year-old man presents to his primary care physician with a 2-week history of what the patient describes as hives on 813
29. A 61-year-old white woman is brought to the ED in critical condition following a motor vehicle accident. On physical exam 813
30. A 67-year-old white man is brought to the ED from his primary care physician's office with decreased urine output, hemopt 814
31. A 68-year-old man presents to his primary care physician with worsening dysphagia and odynophagia. The patient states tha 814
32. A 32-year-old woman undergoes a kidney transplantation, and 2weeks after surgery, she develops stomach pain, diarrhea, we 814
33. An 83-year-old white man is brought to his primary care physician by his daughter for a newly discovered skin lesion. The 815
34. A 22-year-old woman presents to her primary care physician with complaints of fatigue and dyspnea on exertion for the pas 815
35. A 45-year-old premenopausal obese African-American woman presents to her gynecologist with complaints of menorrhagia and 815
36. A 59-year-old white man with a history of hypertension and hyperlipidemia presents to the ED with severe chest pain that 815
37. A 14-year-old African-American boy has a history of splenomegaly and undergoes a splenectomy. A section of his spleen is 816
38. An 18-year-old woman presents to her primary care physician with complaints of diarrhea, weight loss (5lb), and abdominal 816
39. A 30-year-old woman from the northeastern part of the United States presents to her primary care physician with crampy ri 816
40. A 45-year-old homeless man is brought to the ED with vomiting and fever. On physical examination, he is disheveled and ha 816
41. A 35-year-old Hawaiian man presents to his primary care physician for his yearly physical. The only complaint that he has 817
42. A 70-year-old retired man, who worked in a ceramics manufacturing facility, presents to his primary care physician with w 817
43. A 51-year-old woman presents to her primary care physician with weight gain of 10lb in the last month, along with fatigue 817
44. A 4-year-old child presents to the ED with respiratory distress. He had functioned normally at birth until 6months of age 818
45. A 44-year-old woman presents to her physician with arthralgias, fatigue, malaise, and fevers. She also notes that within 818
46. A 40-year-old woman has chronic back pain secondary to an automobile accident 5years earlier that has been worsening for 819
47. A 72-year-old man presented to his primary care physician complaining of a weight loss of 10lb in 1month and a feeling of 819
48. A 30-year-old woman presents to her primary care physician with weight loss, palpitations, and sweating. On physical exam 819
49. A 54-year-old woman is brought to the ED by ambulance with acute SOB. The patient states (through labored breathing) that 820
50. A 64-year-old white man presents to his primary care physician with complaints of four episodes of hematuria over the pas 820
Chapter 28: ECG 821
1. An ECG recorded in a 30-year-old healthy woman is shown in Figure 28-1. How would you interpret this ECG? 821
2. An ECG recorded in a febrile septic patient is shown in Figure 28-2. How would you interpret this ECG? 821
3. An ECG recorded in a healthy middle-aged man after he became dizzy and diaphoretic while having blood drawn is shown in Fi 822
4. The patient whose ECG is shown in Figure 28-4 will almost certainly remain asymptomatic throughout life but is at marg 822
5. The ECG shown in Figure28-5 comes from a 52-year-old man with chronic obstructive pulmonary disease (COPD) follow 823
6. An ECG recorded in a patient with sudden onset of palpitations and rapid heart rate is shown in Figure 28-6. How would 823
7. An ECG in a 38-year-old woman experiencing severe nausea is shown in Figure 28-7. How would you interpret this ECG? 824
8. An ECG recorded in an asymptomatic middle-aged man is shown in Figure 28-8. How would you interpret this ECG? Does h 824
9. An ECG recorded in an elderly woman is shown in Figure 28-9. How would you interpret this ECG? Does she require a pace 825
10. An ECG recorded in a 74-year-old man with recurrent syncope is shown in Figure 28-10. How would you interpret this ECG 825
11. Figure28-11 shows an ECG recorded in a 55-year-old man with a history of a massive myocardial infarction 5 years 826
12. An ECG from a middle-aged man started on procainamide for atrial fibrillation is shown in Figure 28-12. How would yo 826
13. Figure28-13 shows an ECG recorded in a 32-year-old otherwise healthy female smoker who takes birth control pills 826
14. Figure28-14 shows an ECG recorded in a 52-year-old diabetic smoker who is experiencing sudden onset of chest pain 827
15. An ECG recorded in a 75-year-old man with a history of coronary artery disease who is experiencing severe substernal ches 827
16. Figure28-16 shows an ECG from a 42-year-old obese man who is experiencing severe chest pain. How would you interp 827
Index 829
Color Plates 865